Você está na página 1de 51

Imersao Olmpica

Introducao `a Teoria dos Numeros


Eduardo Tengan (ICMC-USP)
20 a 23 de janeiro de 2009

Captulo 1

Divisibilidade e Congru
encias
Neste captulo, introduzimos dois conceitos fundamentais em Teoria dos N
umeros que serao utilizados ao
longo de todo o livro: a relacao de divisibilidade e a de congruencia modulo um n
umero inteiro.

1.1

Divisibilidade

Um dos conceitos primordiais em Teoria dos N


umeros e o da divisibilidade.
Defini
c
ao 1. Dados inteiros dois inteiros d e a, dizemos que
d divide a
ou
d e um divisor de a
ou
a e um m
ultiplo de d
e escrevemos
d|a
se existe um inteiro q tal que a = dq. Em outras palavras, se d 6= 0, dizer que d | a e o mesmo que dizer que
a fraca
o ad e um inteiro.
Se d n
ao divide a (equivalentemente, d n
ao e um divisor de a ou a n
ao e um m
ultiplo de d), escrevemos
d a.
Exemplo 2. Temos que
6 | 12 mas 12 6;
d | 4 se, e somente se, d {1, 2, 4};
Um n
umero a e par se, e somente se, 2 | a;
d | 0, d | d e d | d para todo inteiro d;
0 | 0, j
a que existe um inteiro q tal que 0 = 0 q (qualquer inteiro q serve), muito embora a fraca
o
n
ao esteja definida.
As duas propriedades mais importantes da divisibilidade s
ao as seguintes:
Teorema 3.

1. (d menor) Se a 6= 0 ent
ao
d | a = |d| |a|
1

0
0

2. (d divide) Dados dois m


ultiplos a e b de d, qualquer combinaca
o Z-linear de a e b (i.e., uma express
ao
da forma ax + by com x, y Z) e um m
ultiplo de d. Em smbolos:
(
d|a
= d | (ax + by)
d|b
Demonstraca
o. Se d | a temos que existe um inteiro q tal que a = dq. Por outro lado, como a 6= 0, temos
tambem que q 6= 0, ou seja, |q| 1. Logo
a = dq = |a| = |d| |q| |d| 1 = |d|
o que prova que |a| |d|.
Para provar o d divide, observe que como a e b sao m
ultiplos de d, existem inteiros q1 e q2 tais que
a = dq1 e b = dq2 . Assim, ax + by = d (q1 x + q2 y) e como q1 x + q2 y e inteiro, temos que ax + by e tambem
um m
ultiplo de d.
Os exemplos a seguir ilustram como aplicar estas duas propriedades para resolver problemas de divisibilidade:
Exemplo 4. Encontre todos os inteiros positivos a e b tais que
1 1
1
+ =
a b
6
Soluca
o. Isolando a variavel a (por exemplo) temos
1 1
1
b6
6b
1
=

=
a =
a
6 b
a
6b
b6

Como queremos a > 0, devemos ter b > 6. Assim, o problema e equivalente a encontrar todos os valores
inteiros b 7 para os quais b 6 | 6b. Utilizaremos para isto o d divide (neste caso o b 6 divide) para
simplificar o 6b, reduzindo-o a uma constante:
(
b 6 | 6b
= b 6 | 6b 1 + (b 6) (6) b 6 | 36
b6|b6
Assim, como b 7, temos que b 6 e um divisor positivo de 36, isto e, b 6 {1, 2, 3, 4, 6, 9, 12, 18, 36}.
6b
Para cada um destes valores, obtemos uma solucao com a = b6
inteiro, de modo que as solucoes (a, b) sao
(42, 7) (24, 8) (18, 9) (15, 10) (12, 12) (10, 15) (9, 18) (8, 24) (7, 42)
Observe que as solucoes obtidas sao simetricas em a e b, isto e, se (a, b) e uma solucao ent
ao (b, a) tambem
e uma solucao. Isto n
ao e mera coincidencia: a equacao original ja era simetrica em relacao a a e b, logo
suas solucoes devem manter esta propriedade!
Exemplo 5. Encontre todos os inteiros positivos n tais que
2n2 + 1 | n3 + n2 + 1
Soluca
o. A estrategia aqui e utilizar o d divide para reduzir o grau da expressao n3 + n2 + 1 e em seguida
utilizar o d menor. Basta escolhermos uma combinacao linear conveniente para cancelar o termo de maior
grau n3 :
(
2n2 + 1 | n3 + n2 + 1
= 2n2 + 1 | 2 (n3 + n2 + 1) + (n) (2n2 + 1)
2n2 + 1 | 2n2 + 1

2n2 + 1 | 2n2 n + 2
2

Como o grau de 2n2 n + 2 ainda e maior ou igual ao grau de 2n2 + 1, podemos aplicar mais uma vez o d
divide:
(
2n2 + 1 | 2n2 n + 2
= 2n2 + 1 | (2n2 n + 2) (2n2 + 1)
2n2 + 1 | 2n2 + 1

2n2 + 1 | n + 1

Como o grau de n + 1 e estritamente menor do que o grau de 2n2 + 1, a u


ltima relacao impoe restricoes
severas sobre os possveis valores de n, j
a que para valores suficientemente grandes de n um polinomio de
maior grau possui valor absoluto maior. Assim, utilizando o d menor, temos dois casos a analisar:
n + 1 = 0 n = 1, que e uma possvel solucao, como mostra uma verificacao direta na relacao
de divisibilidade original;
n + 1 6= 0 e portanto 2n2 + 1 | n + 1 implica que
|2n2 + 1| | n + 1| 2n2 + 1 n 1 2n2 n + 2 0
(note que | n + 1| = n 1 pois n > 0 e assim n + 1 0). Esta u
ltima desigualdade n
ao possui
solucoes reais (e muito menos inteiras) ja que o discriminante de 2n2 n + 2 e = 15 < 0.
Logo a u
nica solucao e n = 1.
Exemplo 6. Encontre todos os inteiros a, b 2 para os quais
ab 1
(a 1)(b 1)
e um inteiro.
Soluca
o. Comecamos fazendo uma substituicao psicologica (que embora n
ao altere em nada o problema,
torna-o aparentemente mais f
acil, psicologicamente falando, e claro) cuja finalidade e simplificar o denominador da expressao acima:
(
(
x=a1
a=x+1

y =b1
b=y+1
Assim, o problema e equivalente a encontrar inteiros x, y 1 para os quais a expressao
(x + 1)(y + 1) 1
xy + x + y
x+y
=
=1+
xy
xy
xy
e um inteiro. Em outras palavras, devemos encontrar todos os inteiros positivos x, y tais que xy | x + y.
Como para x e y suficientemente grandes xy e maior do que x + y, novamente nos encontramos na situacao
em que o d menor fornece fortes restricoes para os possveis valores destas variaveis.
Por simetria, podemos nos restringir `
as solucoes em que 1 x y. Neste caso, temos que
xy | x + y = xy x + y 2y = x 2
Assim, temos dois casos:
x = 1 e assim y | y + 1, logo y | (y + 1) y y | 1 y = 1 pelo y divide.
x = 2 e neste caso 2y | y + 2, logo 2y | 2(y + 2) 2y 2y | 4 y | 2 e como y x = 2 a u
nica
possibilidade e y = 2, que claramente e uma solucao.
3

Portanto as solucoes (x, y) sao (1, 1) e (2, 2), isto e, (a, b) = (2, 2) ou (a, b) = (3, 3).
Uma outra maneira de se chegar a estas solucoes e a seguinte: observe que o quociente
1
1
x+y
= + 1+1=2
xy
x y
ja que x, y 1. Logo, sendo este quociente inteiro, devemos ter x+y
xy = 1 ou
podem ser resolvidas utilizando a tecnica do d divide, como no exemplo 4.

1.2

x+y
xy

= 2, equacoes estas que

M
aximo Divisor Comum e Algoritmo de Euclides

Vamos comecar relembrando algumas definicoes que voce provavelmente ja conhece da escola.
Defini
c
ao 7. Se a e b s
ao inteiros, com a 6= 0 ou b 6= 0, denotamos por
def

(a, b) = max{d tais que d | a e d | b}


(ou mdc(a, b) por enfase) o maior divisor comum de a e b. Denotamos tambem por
def

mmc(a, b) = min{m tais que a | m e b | m}


o menor m
ultiplo comum positivo de a e b.
Note que n
ao definimos (0, 0) pois o conjunto dos divisores comuns de 0 e 0 e o Z, logo n
ao h
a maior
divisor comum!
Exemplo 8. Vejamos alguns exemplos.
Temos que (9, 12) = 3, pois 3 e o maior dentre os divisores comuns de 9 e 12:
{1, 3, 9} {1, 2, 3, 4, 6, 12} =
{z
} |
{z
}
|
divisores de 9

divisores de 12

{1, 3}
| {z }

divisores comuns

Da mesma forma, temos que mmc(9, 12) = 36. Note que mdc(9, 12) mmc(9, 12) = 9 12. O fato que
isto n
ao e uma coincidencia ser
a provado mais adiante no pr
oximo captulo.
temos (a, b) = (b, a) > 0 para a e b n
ao simultaneamente iguais a 0, pois se d e um divisor comum de
a e b, ele e um divisor comum de b e a (duh!) e, alem disso, d tambem e um divisor comum, sendo
que ou d > 0 ou d > 0;
para qualquer a 6= 0 temos (a, 0) = |a|, pois os divisores comuns de a e 0 s
ao simplesmente os divisores
de a, dentre os quais |a| e o maior.
Defini
c
ao 9. Se (a, b) = 1, dizemos que a e b s
ao primos entre si. Em outras palavras, a e b s
ao primos
entre si se eles n
ao admitem divisores positivos comuns alem do 1.


N
ao confunda esta definica
o com a noca
o de n
umero primo, aqui a e b n
ao precisam ser n
umeros
primos!
Ja que tocamos no assunto, vamos aproveitar para definir

Defini
c
ao 10. Um inteiro positivo p e chamado de primo se ele possui exatamente dois divisores positivos,
a saber 1 e p. Um inteiro positvo n e chamado de composto se ele pode ser escrito como produto n = ab de
dois inteiros a e b com 1 < a, b < n.
4

Ao contr
ario da crenca popular, 1 n
ao e primo. Ele tambem n
ao e composto. Ent
ao o que ele e?
Ele e uma unidade, ora bolas! Embora possa parecer arbitr
ario a
` primeira vista, esta convenca
o
se mostra bastante razo
avel na pr
atica pois caso admitssemos 1 como primo, v
arios teoremas sobre primos
teriam que ser enunciados como . . . v
alido para todo primo exceto 1. . .
Exemplo 11. Alguns exemplos.
Os primeiros 10 primos s
ao: 2, 3, 5, 7, 11, 13, 17, 19, 23, 29. Uma tabela com os primeiros 240 primos (e
suas razes primitivas) encontra-se no final do livro;
12 e 35 s
ao primos entre si, embora nenhum destes n
umeros seja primo;
se p e um n
umero primo, como os u
nicos divisores positivos de p s
ao 1 e p, temos que
(
1 se p a
(p, a) =
p se p | a
dois n
umeros consecutivos s
ao sempre primos entre si: pelo d divide, um divisor comum d > 0 de n
e n + 1 necessariamente divide 1 = (n + 1) n, logo d = 1;
ao primos entre si, pois se houvesse um divisor comum e > 1
temos que se d = (a, b) ent
ao ad e db s
destes dois n
umeros ent
ao ed seria um divisor comum de a e b estritamente maior do que d.
Agora vamos aplicar o que acabamos de ver para resolver um problema historico: a primeira quest
ao da
prova da primeira Olimpda Internacional de Matem
atica!
Exemplo 12 (IMO). Mostre que a fraca
o

21n + 4
14n + 3

e irredutvel para todo n natural.


Soluca
o. A fracao acima e redutvel se existe um d > 1 que divide simultaneamente o numerador e o
denominador. Assim, mostrar que a fracao e irredutvel e o mesmo que mostrar que qualquer inteiro positivo
d que divide simultaneamente 21n + 4 e 14n + 3 deve ser igual a 1, em outras palavras, devemos mostrar que
21n + 4 e 14n + 3 sao primos entre si para qualquer n natural. Basta utilizar o nosso velho canivete suco, o
d divide:
(
d | 21n + 4
= d | (2) (21n + 4) + 3 (14n + 3) d | 1
d | 14n + 3
Como d > 0, devemos ter d = 1, como queramos.
Exemplo 13. Mostre que 216 + 1 e 232 + 1 s
ao primos entre si.
Soluca
o. Seja d um divisor comum positivo deste dois n
umeros. A ideia natural e tentar utilizar o d
divide, mas a dificuldade e encontrar a combinacao linear correta. Imagine 216 + 1 e 232 + 1 como sendo
polinomios na variavel 2 e tentemos reduzir o grau de 2, como no exemplo 5:
(
d | 216 + 1
= d | 216 (216 + 1) (232 + 1) d | 216 1
d | 232 + 1
Como d | 216 + 1 e d | 216 1 temos que d divide a diferenca 2, logo temos em princpio duas possibilidades:
d = 1 ou d = 2. Porem d n
ao pode ser igual a 2, pois 2 n
ao divide o n
umero mpar 216 + 1. Logo d = 1,
como queramos.
5

Calcular o mdc a partir da definicao e uma tarefa inconveniente para n


umeros grandes (nem t
ao grandes
assim, como voce j
a deve ter percebido). Felizmente h
a um algoritmo bastante eficiente para seu c
alculo,
baseado no algoritmo da divisao que todos n
os estudamos no prim
ario! Destes saudosos tempos, sabemos
que dados dois inteiros positivos a e b, o algoritmo da divisao
b
q

a
r
produz dois inteiros q (quociente) e r (resto) tais que
a=bq+r

0r<b

Para quaisquer dois inteiros a e b (possivelmente negativos) com b 6= 0, podemos estender a nocao de
quociente q e resto r da divisao de a por b utilizando relacoes analogas `as acima:
Defini
c
ao 14. Dados dois inteiros a e b com b 6= 0, o quociente q e o resto r s
ao os u
nicos dois inteiros
determinados pelas relaco
es
a= bq+r
e
0 r < |b|
()
Denotamos o resto da divis
ao de a por b atraves da notaca
o a mod b.
Exemplo 15. Temos a seguinte tabela para a = 13 e b = 3:
a
13
13
13
13

b
3
3
3
3

quociente q
4
5
4
5

resto r = a mod b
1
2
1
2

Para que a definicao acima realmente faca sentido, devemos mostrar


Lema 16. Para quaisquer a e b com b 6= 0, os inteiros q e r na definica
o () acima existem e est
ao
unicamente determinados.
Demonstraca
o. Para mostrar a existencia, considere o conjunto


. . . , 3|b|, 2|b|, |b|, 0, |b|, 2|b|, 3|b|, . . .

dos m
ultiplos de |b| (que e igual ao conjunto dos m
ultiplos de b). O inteiro a encontra-se entre dois m
ultiplos
consecutivos de |b|, digamos k |b| a < (k + 1) |b|. Defina
(
k se b > 0
def
def
q =
e
r = a k |b| = a b q
k se b < 0
Assim, temos a = b q + r e como k |b| a < (k + 1) |b| = 0 a k |b| < |b| temos tambem 0 r < |b|,
como desejado.
Para demonstrar a unicidade, suponha que exista um outro par de inteiros q e r satisfazendo a = bq +r
e 0 r < |b|. Assim,
b q + r = a = b q + r b (q q ) = r r

e portanto r r e um m
ultiplo de b. Porem, como 0 r, r < |b|, temos que |r r| < |b|. Mas o u
nico
m
ultiplo de b com valor absoluto estritamente menor do que |b| e 0, logo r r = 0, ou seja, r = r, e como
() determina q unicamente uma vez conhecidos a, b, r, temos que q = q , completando a demonstracao.

Agora sim estamos prontos para descrever o algoritmo para o c


alculo do mdc, o chamado algoritmo de
Euclides ou algoritmo das divis
oes sucessivas:
6

Teorema 17 (Algoritmo de Euclides). Sejam a e b inteiros com b 6= 0. Ent


ao
(a, b) = (b, a mod b)
Demonstraca
o. Sejam q e r = a mod b o quociente e o resto da divisao de a por b. A fim de provar a
igualdade dos mdcs acima, basta mostrar que os pares a, b e b, r possuem o mesmo conjunto de divisores,
pois em particular o maior deles sera igual. Assim devemos mostrar que
(
(
d|a
d|b

d|b
d|r
Basta utilizar o d divide duas vezes:
(=) Se d divide a e b ent
ao d tambem divide r = a bq.
(=) Se d divide b e r ent
ao d tambem divide a = bq + r.
Exemplo 18. Aplicando reiteradamente o teorema acima, temos um procedimento eficiente para calcular o
mdc de dois inteiros grandes. Por exemplo, temos
(27889, 18937) =
=

1.3

(18937, 8952) = (8952, 1033) = (1033, 688) = (688, 345)


(345, 343) = (343, 2) = (2, 1) = (1, 0) = 1

O anel Z/mZ dos inteiros m


odulo m

No estudo da divisibilidade por um inteiro fixado m, e conveniente classificar todos os inteiros segundo os
seus restos na divisao euclidiana por m, uma especie de taxonomia de Z. Para isto introduzimos a seguinte
Defini
c
ao 19. Seja m um inteiro fixado. Para um inteiro a qualquer, definimos a classe de congruencia de
a m
odulo m, denotada por a, como sendo o conjunto de todos os inteiros b que deixam o mesmo resto que
a na divis
ao euclidiana por m:
a

def

{b Z | b mod m = a mod m}

=
=

{mq + a | q Z}
{b Z | b a e m
ultiplo de m}

Mostremos que as varias definicoes de a acima sao, de fato, todas equivalentes entre si. Escreva a = mq0 +r
com r = a mod m. Note que os inteiros b que deixam resto r sao exatamente os da forma b = mq + r com
q Z. Assim, b = mq +(amq0 ) = m(q q0 )+a e quando q percorre todos os inteiros, q = q q0 tambem
percorre todos os inteiros (lembre-se de que q0 e uma constante determinada por a, a saber o quociente de a
por m). Isto mostra a igualdade dos dois conjuntos da primeira e segunda linhas, enquanto que a igualdade
entre os conjuntos da segunda e terceira linhas e imediata.
Exemplo 20.

se m = 2, temos
0

{. . . , 4, 2, 0, 2, 4, . . .}
{. . . , 3, 1, 1, 3, 5, . . .}

Assim, temos uma partica


o dos inteiros Z em duas classes, 0 (popularmente conhecidos como pares)
e 1 (vulgo mpares). Assim, no que tange a
` divisibilidade por m = 2, h
a somente dois tipos ou
classes de inteiros;
7

No exemplo anterior, temos 0 = 4 = (2) e 1 = 3 = (101), de modo que poderamos indistintamente


escrever a partica
o de Z acima como Z = 0 1 ou Z = 4 (101), por exemplo;
para m = 3, h
a tres tipos ou classes de inteiros m
odulo 3: os que deixam resto 0, os que deixam
resto 1 e os que deixam resto 2 na divis
ao euclidiana por 3.
0 =
1 =
2 =

{. . . , 3, 0, 3, 6, . . .}
{. . . , 2, 1, 4, 7, . . .}
{. . . , 1, 2, 5, 8, . . .}

Em geral, h
a precisamente m classes de inteiros m
odulo m, agrupados nos conjuntos 0, 1, 2, . . . , m 1
segundo os m possveis restos 0, 1, 2, . . . , m 1 na divis
ao euclidiana por m;
note que, para qualquer m e a, o conjunto a e uma progress
ao aritmetica de raz
ao m (infinita para
ambos os lados) que contem a;
Um fato not
avel, e que e a raz
ao pela qual introduzimos a definicao acima, e a compatibilidade das
operacoes aritmeticas de soma, diferenca e produto com a particao de Z em classes modulo m:
Z = 0 1 2 m 1
Para entender o que isto significa, vejamos um exemplo para m = 5. Neste caso as classes modulo 5 sao
0
1
2
3
4

= {. . . , 10, 5, 0, 5, 10, . . .}
= {. . . , 9, 4, 1, 6, 11, . . .}
= {. . . , 8, 3, 2, 7, 12, . . .}
= {. . . , 7, 2, 3, 8, 13, . . .}
= {. . . , 6, 1, 4, 9, 14, . . .}

Veja que se somarmos um elemento de 2 com outro de 4 sempre obtemos um elemento de 2 + 4 = 6 = 1.


Por exemplo, tome 7 2 e 1 4, temos que 7 + (1) = 6 1. Ou 12 2 e 4 4, temos 12 + 4 = 16 1.
Ou ainda 3 2 e 1 4, temos (3) + (1) = 4 1. Em outras palavras, a particao acima e compatvel
com a soma.
Mais ainda, ela tambem e compatvel com o produto: por exemplo, multiplicando um elemento de 2 com
outro de 4 sempre obtemos um elemento de 2 4 = 8 = 3. Vamos fazer um experimento: tome 3 2 e
9 4, temos que (3) 9 = 27 3 (uau, n
ao e que funciona mesmo!).
Chega de experimentos; e hora de provarmos
Lema 21 (Compatibilidade com operacoes aritmeticas). Fixe um inteiro m. Para quaisquer dois elementos
ia

jb

pertencentes a
`s classes de a e b m
odulo m, temos que
i+j a+b
Em outras palavras, temos
(

i=a
j=b

ij ab

ij ab

i + j = a + b
= i j = a b

ij = ab
8

Demonstraca
o. Comecamos observando que pela definicao
(
(
m|ia
ia

jb
m|jb

()

Logo, pelo m divide,


m | (i a) + (j b) m | (i + j) (a + b) i + j a + b
Analogamente mostra-se que i j a b. Finalmente, novamente pelo m divide temos que () implica
m | j (i a) + a (j b) m | i j a b i j a b
O lema acima permite definir a operacoes com as classes da seguinte forma. Por exemplo, a soma de
duas classes a e b e feita tomando-se qualquer elemento i de a, qualquer elemento j de b e definindo a soma
a + b como sendo a classe que contem i + j. O lema nos garante que esta definicao n
ao depende de quais
representantes de classe i e j foram escolhidos. Podemos, por exemplo, tomar i = a e j = b, e neste caso
as definicoes podem ser descritas sucintamente atraves das equacoes
def

def

a + b = a + b,

a b = a b,

def

ab = ab

Em particular, observe que para n natural temos


def

n a = a+ a + + a = n a
|
{z
}
n vezes

def

(a)n = |a a {z. . . a} = (an )


n vezes

Definimos tambem

def

a = (a)

de modo que (a) + a = a + (a) = 0.


Hora de fazer contas! Mas com isso voce ja contava. . .

Exemplo 22. Temos as seguintes tabelas de adica


o e multiplicaca
o para as classes m
odulo m = 6:
+

0
1
2
3
4
5

0
1
2
3
4
5

1
2
3
4
5
0

2
3
4
5
0
1

3
4
5
0
1
2

4
5
0
1
2
3

5
0
1
2
3
4

0
1
2
3
4
5

0
0
0
0
0
0

0
1
2
3
4
5

0
2
4
0
2
4

0
3
0
3
0
3

0
4
2
0
4
2

0
5
4
3
2
1

Observe que estas operaco


es nada mais s
ao do que as operaco
es usuais nos inteiros, somente que elas s
ao
realizadas no ciclo de perodo m = 6:
2

0
4

5
9

Por exemplo, para somar 4 com 3, fazemos a operaca


o em inteiros 4 + 3 = 7 e depois percorremos o ciclo a
partir do 0 em 7 unidades, obtendo o resultado 1. Naturalmente, isto nada mais e do que uma interpretaca
o
pict
oria da igualdade 7 = 1, que corresponde a tomar o resto de 7 na divis
ao por 6.
Exemplo 23. Calcule, em funca
o do n
umero natural n, o resto de 2n na divis
ao por 31.
Soluca
o. Em outras palavras, devemos encontrar, em funcao de n, a classe de congruencia de 2n modulo 31.
n
n1
, podemos calcular estas potencias recursivamente, aproveitando o valor anterior:
Como 2n = 2 = 2 2
0

2
1
2
2
2
3
2
4
2

=
=
=
=
=

1
21
22
24
28

=
=
=
=

2
6
2
7
2
8
2
9
2

2
4
8
16

= 2 16
= 21
= 22
= 24
= 28

= 32 =
= 2
= 4
= 8
= 16

1
...

E a, percebeu um padr
ao? A partir da repeticao 2 = 2 , estabeleceu-se um ciclo de perodo 5. Assim, para
todos os n divisveis por 5 temos 2n = 1, para todos os n da forma n = 5k + 1 (ou seja, aqueles que deixam
resto 1 na divisao por 5) temos 2n = 2, etc. Portanto a resposta final e

1 se n e da forma n = 5k

2 se n e da forma n = 5k + 1
2n mod 31 = 4 se n e da forma n = 5k + 2

8 se n e da forma n = 5k + 3

16 se n e da forma n = 5k + 4
Exemplo 24. Calcule o resto da divis
ao de 122001 por 97.

Demonstraca
o. Nossa missao aqui e determinar a qual das classes de congruencia 0, 1, 2, , 96 modulo 97
o n
umero 121000 pertence. A estrategia e ir fazendo as contas aos pouquinhos, sempre reduzindo o
resultado.
Temos que uma potencia de 12 pr
oxima de 97 e 122 = 144. Assim,
2

12 = 144 = 47
Elevando ao quadrado, temos

12 = 47 = 2209 = 75
Muito bom! Ja sabemos que 124 deixa resto 75 na divisao por 97 sem precisar calcular 124 explicitamente!
Esta ideia parece promissora. Vamos tentar mais uma vez:
8

12

75

(22)2

484 =

96

truque do complementar

Aqui utilizamos um artifcio para simplificar as contas: em vez de utilizar o representante 75 75,
e mais facil calcular o quadrado do igualmente valido representante 22 75 (a quem apelidamos de
8
complementar de 75 modulo 97 j
a que 22 + 75 = 97). Mas agora, pelo mesmo truque, temos 12 = 96 = 1
e nada mais f
acil do que calcular potencias de 1! (bem, calcular potencias de 1 e um pouco mais facil mas
enfim, como diz o velho ditado, 1 dado n
ao se olha nos dentes. . . )
8
2000
Agora rapidamente terminamos o problema: elevando 12 = 1 a 250, obtemos 12
= 1. Finalmente
2001
= 12, isto e, 122001 mod 97 = 12.
multiplicando por 12, obtemos a resposta final 12
10

O proximo exemplo mostra como a utilizacao de classes de congruencia pode reduzir um problema a
analisar apenas um n
umero finito de casos.
Exemplo 25. Mostre que n5 n e um m
ultiplo de 5 para qualquer inteiro n.
Soluca
o. Mostrar que n5 n e m
ultiplo de 5 e mostrar que n5 n est
a na classe de congruencia do 0, isto
e, devemos mostrar que, para qualquer n, n5 n = 0. Temos
n5 n = (n5 ) n = (n)5 n
a um n
umero finito de possibilidades, a saber 0, 1, 2, 3, 4. Assim, o problema, que antes
Mas para n so h
dispunha sobre um n
umero infinito de possveis valores para n, agora se reduz a um n
umero finito de
verificacoes! Temos (utilizando o truque do complementar 3 = 2 e 4 = 1 para simplificar as contas)
n=0
n=1

= (n)5 n = (0)5 0 = 0 0 = 0
= (n)5 n = (1)5 1 = 1 1 = 0

n = 2 = (n)5 n = (2)5 2 = 32 2 = 2 2 = 0

n = 2 = (n)5 n = (2)5 (2) = (2)5 2 = 0

n = 1 = (n)5 n = (1)5 (1) = (1)5 1 = 0

Logo n5 n = (n)5 n e sempre igual a 0, independentemente do valor de n, como desejado.


Terminamos esta secao com uma pequena
Defini
c
ao 26. Fixado m, o conjunto
{0, 1, 2, . . . , m 1}
de todas as classes de congruencia m
odulo m, juntamente com as operaco
es de soma e produto acima definidos, e chamado de anel de inteiros modulo m, e e denotado por Z/mZ ou simplesmente Z/m.


Em Matem
atica, anel e o nome dado a qualquer conjunto A com duas operacoes binarias + (soma) e
(produto) satisfazendo axiomas que abstraem as propriedades usuais das mesmas operacoes sobre
os inteiros (por exemplo). Estes axiomas sao
(associatividade da soma) (a + b) + c = a + (b + c) para todo a, b, c A;
(elemento neutro da soma) existe um elemento 0 A tal que a + 0 = 0 + a = a para todo a A;
(inverso da soma) para todo a A existe um b A tal que a + b = b + a = 0;
(comutatividade da soma) a + b = b + a para todo a, b A;
(associatividade do produto) (a b) c = a (b c) para todo a, b, c A;
(elemento neutro do produto) existe um elemento 1 A tal que 1 a = a 1 = a para todo a A;
(distributividade) a (b + c) = a b + a c e (a + b) c = a c + b c para todo a, b, c A.
Um outro exemplo de anel que voce j
a pode ter visto e o anel de todas as matrizes n n com entradas
reais, com a soma e o produto usual de matrizes. Aqui, os elementos neutros da soma e do produto sao as
matrizes nula e identidade, respectivamente. Note que o produto neste anel n
ao e comutativo.

11

1.4

A Relac
ao de Congru
encia m
odulo m

A notacao a para a classe de congruencia de a modulo m possui o inconveniente de que o modulo m


n
ao aparece explicitamente. Alem disso, como vimos nos problemas acima, em muitas quest
oes estamos
interessados em apenas decidir se a = b para duas classes a e b. Por isso introduzimos a seguinte
Defini
c
ao 27. Dizemos que a e congruente a b modulo m e escrevemos
a b (mod m)
se, e somente se, a e b pertencem a
` mesma classe de congruencia m
odulo m, isto e, se e somente se,
a b (mod m)

a=b
a mod m = b mod m
a = mq + b para algum q
m | (a b)

Em particular, se r = a mod m e o resto da divis


ao de a por m, ent
ao a r (mod m).
Exemplo 28. Temos que
odulo
35 11 (mod 12) pois 12 | 35 11 (em outras palavras, as classes 35 e 11 de congruencia m
12 s
ao iguais). Da mesma forma, 5 19 (mod 12) pois 5 (19) = 24 e um m
ultiplo de 12 ou,
equivalentemente, 5 e 19 deixam o mesmo resto 5 quando divididos por 12;
m | a a = 0 a 0 (mod m).
A notacao de congruencia foi introduzida pelo grande matematico alem
ao Johann Carl Friedrich Gau
(17771855) em sua obra Disquisitiones Arithmeticae (caso seu Latim esteja enferrujado, a traducao para o
Portugues e Indagaco
es Aritmeticas). A definicao de congruencias aparece logo no primeiro par
agrafo desta
importante obra:
Si numerus a numerorum b, c differentiam metitur, b et c secundum a congrui dicuntur, sin
minus, ingongrui; ipsum a modulum appellamus. Uterque numerorum b, c priori in casu alterius
residuum, in posteriori vero nonresiduum vocatur. 1
A relacao de congruencia tem propriedades muito semelhantes `as da relacao de igualdade: quando escrevemos a b (mod m), como a = b, em termos de quest
oes de divisibilidade por m os inteiros a e b sao por
assim dizer equivalentes. Matem
aticos gostam de expressar este fato dizendo que a relacao de congruencia
e uma relaca
o de equivalencia.
Defini
c
ao 29. Seja X um conjunto e uma relaca
o bin
aria em X. Dizemos que e uma relacao de
equivalencia se ela satisfaz os seguintes 3 axiomas:
1. (Reflexividade) a a para todo a X.
2. (Simetria) se a b ent
ao b a.
3. (Transitividade) se a b e b c ent
ao a c.
1 Tradu
ca
o livre: Se o n
umero a divide a diferenca de dois n
umeros b e c, b e c s
ao ditos congruentes com relac
ao `
a a, sen
ao
incongruentes; a
e chamado m
odulo. Ambos os n
umeros b e c s
ao chamados de resduos no primeiro caso e de n
ao resduos no
segundo.

12

Por exemplo, a relacao de igualdade em Z e de equivalencia. Outro exemplo: se X e o conjunto de objetos


de uma sala, ent
ao a relacao
x y o objeto x tem a mesma cor do objeto y
e uma relacao de equivalencia (aqui estamos supondo que todos os objetos sao monocrom
aticos, ent
ao para
n
os n
ao existe a mesa verde com pes azuis!). Ainda um terceiro exemplo: se X e o conjunto de cartas de
um baralho, defina x y se, e somente se, as duas cartas tem o mesmo naipe.
Os exemplos acima sao casos particulares do seguinte fato: uma particao de X em conjuntos (dois a dois
disjuntos) Xi define uma relacao de equivalencia em X via x y x e y pertencem ao mesmo conjunto
Xi . No primeiro exemplo, a relacao de igualdade corresponde `a particao de Z em conjuntos unit
arios:
Z = {2} {1} {0} {1} {2}
No segundo exemplo, temos a particao dos objetos segundo suas cores. E no terceiro exemplo, a particao
do baralho em 4 conjuntos segundo seus naipes. Como a relacao de congruencia e definida por a b
(mod m) a e b pertencem `
a mesma classe de congruencia modulo m, temos que e a relacao de
equivalencia em Z correspondente `
a particao
Z = 0 1 2 m 1
O fato da relacao de congruencia ser de equivalencia e interessante, mas ate a nada de impressionante,
afinal h
a uma multid
ao de relacoes de equivalencia pelo mundo afora, qualquer particao de Z daria origem
a uma. O que e not
avel, e que foi a grande sacada do Gau, e o fato de que essa relacao de equivalencia e
compatvel com as operacoes aritmeticas usuais. Resumimos esta discuss
ao no seguinte
Teorema 30 (Congruencias: propriedades operacionais).
1. (Relaca
o de equivalencia) Para um m fixado, a relaca
o de congruencia m
odulo m e de equivalencia,
ou seja, satisfaz
(i) (Reflexividade) a a (mod m) para todo a.

(ii) (Simetria) se a b (mod m) ent


ao b a (mod m).

(iii) (Transitividade) se a b (mod m) e b c (mod m) ent


ao a c (mod m).
2. (Compatibilidade com soma, diferenca e produto) Suponha que
ab

(mod m)

cd

(mod m)

Ent
ao
a+c
ac
ac

b + d (mod m)
b d (mod m)

b d (mod m)

(1.1)
(1.2)
(1.3)

Em particular, temos
na
an

nb (mod m)

para todo n inteiro

(1.4)

bn

para todo n natural

(1.5)

(mod m)

3. (Cancelamento) Se (n, m) = 1 ent


ao
na nb (mod m) = a b
ou, equivalentemente,
n a = n b = a = b
13

(mod m)

Mas atenca
o, o cancelamento s
o vale quando (n, m) = 1!

Demonstraca
o. Ja demonstramos 1 e provaremos 3 no proximo captulo (listamos esta propriedade aqui
por praticidade apenas). Finalmente, 2 e apenas a traducao do lemma 21 na relacao de congruencias: por
ao
exemplo, para mostrar 1.1 note que, por hipotese, temos a = b e c = d ent
a+c=a+c=b+d=b+d
e assim a + c b + d (mod m).
As propriedades operacionais nos dizem que a relacao de congruencia modulo m funciona quase como
se fosse a relacao de igualdade, pois podemos somar, subtrair e multiplicar congruencias, e mesmo dividir
em certas situacoes.
Exemplo 31. Mostre que se n e inteiro positivo mpar ent
ao 2n + 1 e um m
ultiplo de 3.
Soluca
o. Temos 2 1 (mod 3). Como n e mpar, elevando a n ambos os lados (propriedade 1.5) obtemos
2n (1)n

(mod 3) 2n 1 (mod 3)

Somando com a congruencia 1 1 (mod 3) (simetria e propriedade 1.1), obtemos


2n + 1 0 (mod 3) 3 | (2n + 1)
que era o que queramos provar. Veja que, na pratica, o que fizemos foi nada mais nada menos do operar
com as congruencias como em uma equacao, elevando os dois lados a um mesmo expoente e passando para
o outro lado o 1, como em uma igualdade.
O exemplo acima ilustra o fato de que, ao calcularmos an mod m, e interessante procurar um expoente
d tal que ad 1 (mod m). No captulo 3 veremos uma formula para tal expoente, mas por hora fica a
mensagem:

Exemplo 32. Calcule o resto da divis


ao de 22002 por 101.

Soluca
o. Aqui, procedemos como em 24. Para calcular 22002 mod 101, comecamos com uma potencia de
2 pr
oxima de 101, por exemplo 27 = 128. Elevando ao quadrado varias vezes e utilizando o truque do
complementar, obtemos
27 27 (mod 101) = 214 729 (mod 101) 214 22 (mod 101)
= 228 484 (mod 101) 228 21 (mod 101)

= 256 441 (mod 101) 256 64 (mod 101)

Mas 64 = 26 , o que nos d


a a oportunidade de aplicar nossa nova ferramenta, o cancelamento:
256 26

(mod 101) = 250 1 (mod 101)

E providencialmente encontramos 250 1 (mod 101)! Agora e facil terminar: dividindo 2002 por 50,
obtemos 2002 = 50 40 + 2 e assim
40 2
2 = 22002 (1)40 22 (mod 101) 22002 4 (mod 101),
22002 = 250

ou seja, 22002 4 (mod 101). Logo o resto da divisao de 22002 por 101 e 4.
14

Exemplo 33. Calcule (3200 1, 335 1).


Soluca
o. A ideia natural para calcular o mdc acima e utilizar o algoritmo de Euclides, que e o que vamos
fazer! Primeiramente temos que
(3200 1, 335 1) = (335 1, 3200 1 mod 335 1)
e assim devemos calcular 3200 1 mod 335 1. Pela filosofia 1 devemos encontar uma potencia de
3 que e congruente a 1 modulo 335 1, mas neste caso temos um candidato natural:
335 1 (mod 335 1)
Dividindo 200 por 35 obtemos 200 = 35 5 + 25, logo
3200 = (335 )5 325

= 3200 15 325 (mod 335 1)


= 3200 1 325 1 (mod 335 1)

Como 0 325 1 < 335 1, temos que 325 1 e o resto da divisao de 3200 1 por 335 1. Agora devemos
calcular
(335 1, 325 1) = (325 1, 335 1 mod 325 1)
Com o mesmo procedimento acima, obtemos apos algumas contas (verifique!) que 335 1 mod 325 1 =
310 1. Assim, aplicando reiteradamente este processo temos
(335 1, 325 1) = (325 1, 310 1) = (310 1, 35 1) = 35 1
pois 310 1 = (35 1)(35 + 1) e m
ultiplo de 35 1. Logo (3200 1, 335 1) = 35 1.
Vejamos um exemplo em que congruencias nos ajudam a provar a inexistencia de solucoes.
Exemplo 34. Mostre que se n 3 (mod 4) ent
ao n n
ao pode ser escrito como soma de dois quadrados
perfeitos.
Soluca
o. Suponha que n = x2 + y 2 . O ponto crucial aqui e notar que n
ao h
a muitos quadrados perfeitos
modulo 4. Analisando todas as possibilidades, temos
x=0

= x2 = 0

x = 1 = x2 = 1
x = 2 = x2 = 0
Assim, um quadrado perfeito so deixa resto 0 ou 1 modulo 4. Portanto
x2 + y 2 mod 4 {0, 1, 2}
Mas como n mod 4 = 3 n
ao est
a no conjunto acima, n
ao podemos ter n = x2 + y 2 .
Terminamos com uma simples
Defini
c
ao 35. Um conjunto de m n
umeros inteiros
a1 , a2 , . . . , am
e chamado de sistema completo de resduos (ou restos) se ai 6 aj (mod m) para i 6= j, ou seja, se os
n
umeros ai representam todos os elementos de Z/mZ exatamente uma vez:
Z/mZ = {a1 , a2 , . . . , an }
15

1.5

Algumas aplicac
oes

Vamos agora mostrar algumas aplicacoes dos conceitos vistos nas secoes anteriores em problemas que aparecem na natureza.

1.5.1

Bases Num
ericas (vulgo Problemas com Dgitos)

Um dos temas mais populares em olimpadas de Matem


atica sao os bons e velhos problemas com dgitos.
O enunciado destes problemas muitas vezes mascaram fatos bem conhecidos da teoria dos N
umeros, ent
ao e
importante familiarizar-se com eles.
Dado um n
umero natural n, podemos expand-lo em base 10 como
n = ad 10d + ad1 10d1 + + a1 10 + a0

()

com dgitos ai {0, 1, 2, . . . , 9}. Por exemplo,

19785 = 1 104 + 9 103 + 7 102 + 8 101 + 5 100

Muitos problemas com dgitos se resumem a escrever a expansao (). Por exemplo, os velhos criterios de
divisibilidade por 9 e 10 nada mais sao do que uma aplicacao imediata do nosso princpio 1 .
Exemplo 36 (criterios de divisibilidade por 9 e 11). Seja
n = ad 10d + ad1 10d1 + + a1 10 + a0

0 ai 9

a expans
ao do n
umero natural n na base 10. Ent
ao
1. n tem o mesmo resto na divis
ao por 9 que a soma de seus dgitos a0 + a1 + + ad . Em particular,
9 | n 9 | (a0 + a1 + + ad );
2. n tem o mesmo resto na divis
ao por 11 que a soma alternada de seus dgitos a0 a1 + a2 ad
(comecando sempre pelo dgito da unidades). Em particular, 11 | n 11 | (a0 a1 + a2 ad ).

Soluca
o. Como 10 1 (mod 9) e 10 1 (mod 11) temos
n
n

ad 1d + ad1 1d1 + + a1 1 + a0 (mod 9)


ad (1)d + ad1 (1)d1 + + a1 (1) + a0

(mod 11)

e o resultado segue.
Um problema bastante comum e o de determinar os
ultimos dgitos de uma potencia grande. Neste
caso, o fato principal a ser utilizado e o seguinte: suponha que voce queira encontrar os dois u
ltimos dgitos
a0 de a1 de n em (). Como
n = (ad2 10d2 + ad3 10d3 + + a3 10 + a2 ) 102 + (10a1 + a0 )

e 0 10a1 + a0 10 9 + 9 < 100, temos que 10a1 + a0 e o resto da divisao de n por 102 = 100, logo o
problema se resume a calcular n mod 100. Caso voce queira os tres u
ltimos dgitos, basta calcular n mod 103
e assim por diante.
Exemplo 37. Determine os dois u
ltimos dgitos de 32009 .
Soluca
o. Vamos calcular 32009 mod 100. Fazendo as contas modulo 100, temos
4

Portanto, de 3

10

10

3 = 19 = 3 = (19)2 = 61 = 3
= 49 obtemos
3

20

= 3 61 = 49

= 49 = 1 = 32000 = 1

2009

Ja calculamos 3 = 61, logo multiplicando por 3 obtemos 3 = 83. Logo 3


u
ltimos dgitos de 32009 sao 83.
16

= 1 3 = 83. Assim, os dois

Em alguns problemas, devemos tambem estimar a quantidade de dgitos de n. Isto e simples: como um
n
umero n tem d dgitos se, e somente se, ele e maior ou igual a 10d1 (menor n
umero com d dgitos) e
estritamente menor do que 10d (menor n
umero com d + 1 dgitos), e como o logartmo na base 10 e uma
funcao crescente, conclumos que
10d1 n < 10d = d 1 log10 n < d = d = log10 n + 1
onde, para um n
umero real x, x denota a parte
nico inteiro t tal que x 1 < t x.
inteira t de x, isto e, o u
Por exemplo, 2,1 = 2, = 3, 3 = 3 e 2 = 2 (nao e 1!). Resumindo
Lema 38 (N
umero de dgitos). Seja n um natural. Ent
ao o n
umero de dgitos de n e igual a log10 n + 1.
Vejamos como aplicar esta ideia juntamente com as anteriores no seguinte
Exemplo 39. (IMO) Seja A a soma dos dgitos de 44444444 e B a soma dos dgitos de A. Ache a soma
dos dgitos de B.
Soluca
o. Este problema pode impressionar `a primeira vista, mas de fato ele e mais simples do que aparenta.
O ponto chave aqui e perceber que a operacao soma dos dgitos fazer encolher tremendamente o seu
argumento!
Por exemplo, embora 44444444 seja gigantesco, como 44444444 < (104 )4444 = 1017776 , temos que 44444444
tem no maximo 17776 algarismos e portanto a soma A de seus dgitos n
ao ultrapassa 17776 9 = 159984,
um n
umero min
usculo quando comparado a 44444444 ! (sem o fatorial, e claro).
Como A 159984, temos que a soma B de seus dgitos n
ao ultrapassa 9 + 9 + 9 + 9 + 9 = 45. De
fato, este e o pior dos casos para A < 105 ; no caso em que 105 A 159999, teramos no maximo
B 1 + 5 + 9 + 9 + 9 + 9 = 42 e 42 < 45.
Assim, B 45 e a soma C dos dgitos de B portanto n
ao ultrapassa 3 + 9 = 12. Mas como encontrar
C exatamente? A entram as congruencias: utilizando o criterio de divisibilidade por 9, sabemos que a
operacao soma dos dgitos n
ao altera a classe de congruencia modulo 9. Como
44444444 (4 + 4 + 4 + 4)4444

(mod 9) 44444444 (2)4444

(mod 9)

e (2)4444 7 (mod 9) (verifique, utilize o fato que (2)3 1 (mod 9)) conclumos que o resto da divisao
de 44444444 por 9, bem como os restos das divisoes de A, B e C por 9, sao todos iguais a 7. Mas 0 < C 12,
logo a u
nica possibilidade e C = 7, que e a soma procurada.

1.5.2

Usando congru
encias para resolver equac
oes diofantinas

As congruencias sao extremamente u


teis na resolucao de equacoes em inteiros. Tais equacoes sao chamadas

de diofantinas, em homenagem ao matematico grego Diofanto de Alexandria (o o `o A


),
que viveu no seculo III D.C. e escreveu uma serie de livros entitulados Arithmetica sobre resolucoes de
equacoes algebricas.
Exemplo 40. (OBM) Determine todos os inteiros positivos m e n tais que
m2 + 161 = 3n
Soluca
o. Comecamos com um caso particular: suponha que n = 2k seja par. Mas por que estudar este
caso primeiro? Porque agora vale a fatoracao
m2 + 161 = 3n 161 = 32k m2 161 = (3k m)(3k + m)
Pois e, fatore e fature! H
a um n
umero finito de maneiras de escrever 161 como o produto de dois inteiros
3k m e 3k + m. Utilizando o fato que 161 = 7 23 e que 3k m < 3k + m temos somente dois casos:
(
(
3k m = 1
3k m = 7
e
k
3 + m = 161
3k + m = 23
17

(na verdade, aqui estamos nos adiantando um pouco porque utilizamos implicitamente o Teorema Fundamental da Aritmetica sobre a fatoracao u
nica em primos, que sera provado no proximo captulo. . . )
No primeiro caso, resolvendo o sistema nas variaveis m e 3k obtemos m = 80 e 3k = 81 k = 4,
ou seja, (m, n) = (80, 8). No segundo caso, temos m = 15 e 3k = 8, que n
ao possui solucoes inteiras.
Ser
a que a solucao acima e a u
nica? Para provar isto, vamos utilizar congruencia para mostrar que n e
obrigatoriamente par. A primeira d
uvida que poderia surgir e que modulo vamos usar? Aqui a ideia e
trabalhar modulo um n
umero que simplifique a equacao acima. Por exemplo, podemos tentar apagar o
161, e para isto escolhemos trabalhar modulo um de seus divisores, por exemplo 7:
m2 + 161 = 3n = m2 + 161 3n

(mod 7) m2 3n

(mod 7)

Em outras palavras, agora temos que comparar os possveis restos de m2 e 3n na divisao por 7. Como no
exemplo 23, as potencias de 3 sao periodicas modulo 7 com perodo 6:
1
3
2
6
4
5

12

13

15

11

= 3 =3 =3

= 3 = 3 = 3 =
2
8
14
= 3 = 3 = 3 =

= 3 = 3 = 3 =
4
10
16
= 3 = 3 = 3 =
= 3 =3

=3

17

Agora veja que h


a poucas possibilidades para os quadrados perfeitos m2 modulo 7; utilizando o truque do
complementar e analisando o n
umero finito de casos, temos
m=0

= m2 = 0

m = 1 = m2 = 1

m = 2 = m2 = 4
m = 3 = m2 = 2

Assim, para que m2 = 3 , devemos ter n = 6k (quando m2 = 1), n = 6k + 4 (quando m2 = 4) ou n = 6k + 2


(quando m2 = 2). Em todos os casos, n e par, logo a solucao que encontramos acima e a u
nica da problema.
Exemplo 41. Encontre todos os n
umeros naturais x e y tais que 2x = 3y 1.
Soluca
o. Aqui, qual o modulo? Voce lembra do 1 ? Pois ele ataca novamente! Podemos, por
exemplo, trabalhar modulo 4, que e proximo de 3 e e uma potencia de 2:
2x = 3y 1 = 2x 3y 1 (mod 4) 2x (1)y 1 (mod 4)
Agora precisamos analisar 2x mod 4. A grande maioria das potencias de 2 e divisvel por 4, mas considerar
todas precisamos dividir em casos:
se x = 0, temos 20 = 3y 1 3y = 2, o que n
ao e possvel;
se x = 1, temos 21 = 3y 1 3y = 3 y = 1, logo x = 1 e y = 1 e uma solucao;
se x 2 (caso geral), temos 2x 0 (mod 4). Logo devemos ter (1)y 1 0 (mod 4), o que ocorre
se, e somente se, y e par. Assim, y = 2a, onde a e natural e voltando na equacao original temos uma
fatoracao:
2x = 32a 1 2x = (3a 1)(3a + 1)
Como 3a 1 e 3a + 1 sao divisores de uma potencia de 2, ent
ao 3a 1 e 3a + 1 sao ambas potencias
de 2 pelo Teorema Fundamental da Aritmetica (cuja demonstracao veremos mais tarde, n
ao perca o
18

proximo captulo!). Mas a diferenca entre elas e (3a + 1) (3a 1) = 2, e as u


nicas potencias de 2 t
ao
pr
oximas assim sao 2 e 4. De fato, olhando para as potencias de 2,
20 = 1, 21 = 2, 22 = 4, 23 = 8, 24 = 16, 25 = 32, . . .
observamos que se i > j e i 3 temos 2i 2j 2i 2i1 = 2i1 22 = 4, logo devemos j i 2 e
agora analisando o n
umero finito de possveis pares (i, j) temos que (i, j) = (1, 2) e a u
nica possibilidade
vi
avel.
Consequentemente, 3a 1 = 2 a = 1 e logo y = 2 e x = 3.
Enfim, as u
nicas solucoes sao (x, y) = (3, 2) e (x, y) = (1, 1).

1.6
1.6.1

Exerccios
B
asicos

1. Encontre todos os inteiros positivos tais que


(a) n + 1 | n3 1
1
1
(c) n1 + m
= 143

(b) 2n 1 | n3 + 1
(d) 2n3 + 5 | n4 + n + 1

2. Mostre que
(a) 31 | 2015 1
(c) 641 | 232 + 1
(e) 7 | 22225555 + 55552222

(b) 61 | 2015 1
(d) 13 | 270 + 370

3. Determine o resto das divis


oes de
(a) 41234 por 3
(c) 2000 22000 1 por 3

(b) 20100 por 17


(d) 520 por 26

4. Encontre os dois u
ltimos dgitos de
(a) 7100
(c) 5600 + 19200
18
5. Mostre que 8355 + 6
1 e divisvel por 112.

(b) 22000
(d) 72000 2300

6. Mostre que se n e um n
umero natural composto, ent
ao n e divisvel por um primo p com p n.
7. Qual o n
umero de dgitos de 20082008 quando escrito na notaca
o decimal? (log10 2008 3.302763 . . .)

1.6.2

Lic
ao de Casa

8. Mostre que para todo natural n


(a) 7 | 23n+2 + 3
(c) 25 | 722n+2 472n + 282n1

(b) 7 | 37n+2 + 16n+1 + 23n


(d) 24 | n(n2 1)(3n + 2)

9. Mostre que
(a) 215 1 e 210 + 1 s
ao primos entre si.
(b) 232 + 1 e 24 + 1 s
ao primos entre si.
10. Seja an = n2 + 100. Determine o maior valor possvel de (an , an+1 ).
11. Encontrar os 4 u
ltimos dgitos de 32008 na notaca
o decimal.
Dica: para reduzir as contas, aplique o bin
omio de Newton em (10 1)1004 .
19

12. Mostre que se m e n s


ao dois inteiros positivos tais que mn + 1 e divisvel por 24 ent
ao m + n tambem
e divisvel por 24.
13. Encontre todos os n
umeros naturais de dois dgitos divisveis pela soma de seus dgitos.
14. Em cada casa do tabuleiro de xadrez, h
a o n
umero de gr
aos de trigo indicado, como mostra a figura. O
cavalo de Bruno & Bernardo comeca a se movimentar no tabuleiro, de acordo com as regras usuais, a partir
de uma casa qualquer. Quando ele atinge uma casa, come todos os gr
aos nela existentes (mas ele n
ao come
os gr
aos da casa inicial). Quando ele deixa uma casa, n
os recolocamos a mesma quantidade de gr
aos que
nela existiam. Depois de um certo tempo, o cavalo de Bruno & Bernardo retorna a
` casa inicial e come os
gr
aos nela existentes. Prove que o n
umero de gr
aos que o cavalo de Bruno & Bernardo comeu durante sua
viagem e divisvel por 3
263 262 261

216 217 218

1.6.3

215 214 213 212 211 210 29

28

20

27

21

22

23

24

25

26

Quest
oes de Provas

15. (IMO) Encontre todos os inteiros a, b, c com 1 < a < b < c tais que (a 1)(b 1)(c 1) e divisor de
abc 1.
16. Encontre os dois u
ltimos dgitos de 22n (22n+1 1) para n 1 mpar.
2009

17. Quais os dois u


ltimos dgitos de 33
18.

(a) Mostre que 81 | 99


. . . 9} .
| {z
nove noves

(b) Determine o menor n


umero da forma 99 . . . 9 que e divisvel por 17.
19. Mostre que 41 divide 11 . . . 1 (onde h
a 5k dgitos 1, k inteiro positivo).
20. Mostre que 91 divide 11 . . . 1 (onde h
a 6k dgitos 1, k inteiro positivo).
21. Mostre que, para todos k, m, n naturais, 55k+1 + 45m+2 + 35n e divisvel por 11.
22. Encontre todos os pares de inteiros positivos (x, y) tais que 2x = 1 + 3y .
23. Encontre todos os inteiros positivos x e y tais que 3x 2y = 7.

20

Captulo 2

Equac
oes Diofantinas Lineares e
Teorema Fundamental da Aritm
etica
Uma equaca
o diofantina linear (em duas variaveis) e uma equacao da forma
ax + by = c
nas variaveis x e y. Diofantina, como voce ja sabe, se refere `a qualquer equacao em n
umeros inteiros,
enquanto que linear obviamente e uma referencia ao fato de que a equacao acima ser a de uma reta no
plano cartesiano.
Assim, resolver uma equacao diofantina linear e o mesmo que encontrar todos os pontos inteiros (isto e,
com ambas as coordenadas inteiras) de uma reta. Por exemplo, tomando a reta
6x + 21y = 2
embora haja uma infinidade de pontos reais (e ate mesmo racionais) n
ao h
a nenhum ponto inteiro! De fato,
se x, y Z, o lado esquerdo e m
ultiplo de 3, enquanto que o direito n
ao.
Neste captulo, vamos aprender a reconhecer quando uma equacao diofantina linear possui ou n
ao solucao
e, em caso afirmativo, como fazer para encontrar todas. Equacoes diofantinas lineares tem muitas aplicacoes
importantes: elas permitem provar, por exemplo, os teoremas do cancelamento para congruencias e o Teorema
Fundamental da Aritmetica sobre a decomposicao u
nica em primos.

2.1

Teorema de B
ezout

O exemplo acima mostra uma obstrucao obvia para a existencia de solucoes inteiras de ax + by = c: se
houver um divisor comum d de a e b que n
ao divide c ent
ao n
ao h
a solucoes inteiras. O teorema de Bezout
diz que, na ausencia de tal obstrucao, a equacao admite solucao.
Teorema 42 (Bezout). A equaca
o
ax + by = c
admite soluca
o se, e somente se, (a, b) | c.
Demonstraca
o. Comecamos com a implicacao trivial: suponha que a equacao admita solucao, isto e, existem
x, y inteiros tais que ax + by = c. Como (a, b) divide o lado esquerdo da equacao conclumos que (a, b) | c
tambem.
Reciprocamente, suponha que (a, b) | c. Basta mostrar que a equacao
ax + by = (a, b)
21

tem solucao, pois multiplicando uma solucao desta equacao por c/(a, b) obtemos uma solucao da equacao
original. Em outras palavras, devemos mostrar que (a, b) escreve-se como uma combinacao linear de a e b.
Dentre todas as combinacoes lineares positivas ax+ by de a e b (existe pelo menos uma, verifique!), seja m
a menor, digamos m = ax0 + by0 . Observe que (a, b) | m pois (a, b) divide a e b e m e uma combinacao linear
de a e b. Vamos mostrar agora que m divide qualquer combinacao linear de a e b. Uma vez provado isto,
temos que, em particular, m e um divisor comum de a e b, logo m (a, b), que juntamente com (a, b) | m,
implica m = (a, b).
Seja t = ax + by uma combinacao linear qualquer de a e b. Seja q e r o quociente e o resto da divisao de
t por m, de modo que t = qm + r com 0 r < m. Temos
r = t qm = (ax + by) q(ax0 + by0 ) = a(x qx0 ) + b(y qy0 )
Ou seja, o resto r tambem e uma combinacao linear de a e b! Como r < m, pela minimalidade do m a u
nica
possibilidade e r = 0, isto e, m | t, como desejado.
A prova do teorema acima n
ao permite encontrar explicitamente uma solucao de ax + by = c caso ela
exista. Porem isto pode ser facilmente feito utilizando-se o algoritmo de Euclides! Veja o
Exemplo 43 (Resolvendo Equacoes Diofantinas Lineares via Euclides). Encontre uma soluca
o inteira de
1001x + 109y = 2.
Soluca
o. Calculamos o mdc (1001, 109) via o algoritmo de Euclides:
1001

109 9 + 20

109

20 5 + 9

20

9 2+ 2

2 4+ 1

Assim, (1001, 109) = (109, 20) = (20, 9) = (9, 2) = (2, 1) = 1 e pelo teorema de Bezout a equacao diofantina
linear acima possui solucao.
Para encontrar uma, devemos expressar 2 como combinacao linear de 1001 e 109. Note que a u
ltima
divisao permite expressar 1 como combinacao linear de 9 e 2:
9 1 2 4 = 1
Mas da pen
ultima divisao, temos que 2 = 20 9 2, logo substituindo esta expressao na combinacao
linear acima temos
9 ( 20 9 2) 4 = 1 9 9 20 4 = 1
e agora expressamos 1 como combinacao linear de 20 e 9! Repetindo este procedimento, eventualmente
expressaremos 1 como combinacao linear de 1001 e 109; multiplicando tudo por 2, obteremos ent
ao nossa
desejada solucao!
Vamos fazer isto de modo mais sistem
atico: isolamos os restos no algoritmo de Euclides e fazemos as
substituicoes, partindo da u
ltima equacao, e tomando o cuidado de lembrar quais sao os coeficientes nas
equacoes durante as simplificacoes:
20

1001 109 9

109 20 5

20 9 2

9 2 4
22

e assim
1

9 ( 20 9 2) 4 = 9 9 20 4

= ( 109 20 5) 9 20 4 = 109 9 20 49

109 9 ( 1001 109 9) 49 = 1001 (49) + 109 450

Portanto (x, y) = (49, 450) e uma solucao de 1001x + 109y = 1, logo multiplicando tudo por 2 temos que
(x, y) = (98, 900) e uma solucao de 1001x + 109y = 2.
O procedimento acima permite dar uma outra demonstracao (da parte n
ao trivial) do teorema de Bezout
via o princpio da induca
o finita (PIF). Vamos esboca-la aqui quando a b > 0 e (a, b) = 1, deixando para
voce completar os detalhes no caso geral. A inducao e sobre min{a, b}.
Dividindo a por b, temos a = bq + r com 0 r < b. Temos que
ax + by = 1 (bq + r)x + by = 1 b(qx + y) + rx = 1
Como (a, b) = (b, r) = 1 e min{b, r} = r < b = min{a, b}, por hipotese de inducao podemos encontrar
inteiros x0 e y0 para os quais bx0 + ry0 = 1. Logo, da equivalencia acima, temos que x = y0 e y = x0 qy0
e uma solucao de ax + by = 1.
O teorema de Bezout n
ao serve so para resolver equacoes diofantinas lineares; ele tem importantes
aplicacoes teoricas, como por exemplo o
Teorema 44 (Alternativa de primo). Se d | ab e (d, a) = 1 ent
ao d | b. Em particular, se p e um n
umero
primo e
p | a 1 a 2 . . . an
ent
ao p | ai para algum i.
Demonstraca
o. Pelo teorema de Bezout, existem x, y inteiros tais que ax + dy = 1. Multiplicando por b
temos
abx + bdy = b
Note d divide tanto abx (pois por hip
otese d | ab) como bdy, logo d | b tambem, provando a primeira parte
do lema.
A segunda parte segue da primeira: se p n
ao divide a1 , ent
ao p | a2 a3 . . . an . Agora, suponha que p n
ao
divida a2 tambem. Teramos que p | a3 a4 . . . an . E assim por diante, de modo que enventualmente p ter
a
que dividir algum ai .
Podemos agora finalmente provar o
Corol
ario 45 (Cancelamento). Se (n, m) = 1 ent
ao
na nb

(mod m) = a b

(mod m)

ou, equivalentemente,
n a = n b = a = b
Demonstraca
o. Note que
na nb (mod m) m | n(a b)
Como (n, m) = 1, pela alternativa de primo temos que
m | a b a b (mod m)
como desejado.
23

Podemos retornar agora `


as equacoes diofantinas lineares. Ja sabemos identificar quando uma equacao
destas possui solucao e ate mesmo encontrar uma. Como fazer para encontrar todas? Veja o
Exemplo 46. Encontre todas as soluco
es de 1001x + 109y = 2.
Soluca
o. Vamos reescrever o lado direito utilizando a solucao particular encontrada no exemplo anterior:
1001x + 109y = 2 = 1001 (98) + 109 900 1001(x + 98) = 109(y 900)
Como 1001 e 109 sao primos entre si, pela alternativa de primo temos que 1001 divide y 900, isto e,
y 900 = 1001t para algum t. Assim, substituindo na equacao acima, temos 1001(x+98) = 1091001t
x + 98 = 109t, e as solucoes sao portanto
x = 98 109t

y = 900 + 1001t

com t Z.
Em outras palavras, os pontos inteiros da reta de equacao 1001x + 109y = 2 sao os pontos da forma
(x, y) = (98, 900) + (109, 1001)t para t Z, que est
ao igualmente espacados na reta, sendo obtidos a
partir da translacao do ponto inteiro (98, 900) por m
ultiplos inteiros do vetor (109, 1001).

2.2

Inverso Multiplicativo

Vimos como o cancelamento permite dividir modulo m sob certas circunst


ancias. Nesta secao, queremos
tornar mais preciso este conceito de divisao modulo m:
Defini
c
ao 47. Dado um elemento a
de Z/mZ, definimos o seu inverso multiplicativo, denotado por a
1 ,
1
como sendo o elemento (caso exista) de Z/mZ tal que a
a
= 1.
1

Por exemplo, como 7 13 1 (mod 15) temos que 7

= 13 em Z/15Z.

Teorema 48. O elemento a


Z/mZ possui inverso multiplicativo se, e somente se, a e primo com m.
Demonstraca
o. Note que o inverso multiplicativo x
de a
existe se, e somente se, a equacao
ax 1 (mod m) ax my = 1 para algum y
possui solucao. E pelo teorema de Bezout, a equacao diofantina linear acima e sol
uvel se, e somente se,
(a, m) = 1.
Assim, encontrar o inverso multiplicativo de a
Z/mZ e o mesmo que resolver a equacao ax my = 1,
1
o que pode ser feito utilizando-se o algoritmo de Euclides. Assim, por exemplo, temos que 109 = 450 em
Z/1001.
De posse do inverso multiplicativo, temos agora todas as ferramentas para realmente tratar congruencias
quase como uma relacao de igualdade. Como exemplo, vamos resolver a seguinte equacao:
Exemplo 49. Determine todos os inteiros x tais que
13x + 8 6x + 11 (mod 15)
Soluca
o. Isolando a variavel x (i.e. subtraindo 6x + 8 de ambos os lados da congruencia) obtemos
13x + 8 6x + 11 (mod 15)

24

13x 6x 11 8 (mod 15)


7x 3 (mod 15)

Agora gostaramos de dividir por 7 modulo 15. Mas isto e facil! Basta multiplicar pelo inverso de 7 em
Z/15Z, ou seja, 13:
7x 3 (mod 15)

13 7x 13 3 (mod 15)
x9

(mod 15)

E voila! Temos a resposta: todos os inteiros x tais que x 9 (mod 15), ou seja, todos os inteiros da forma
x = 9 + 15k para k Z.
Vamos ilustrar os conceitos acima provando o
Teorema 50 (Teorema de Wilson). Um natural p > 1 e primo se, e somente se, (p 1)! 1 (mod p).
Demonstraca
o. Suponha que p seja primo. Note que neste caso todos os elementos de Z/pZ, com excecao
de 0, sao inversves. Devemos provar que o produto (p 1)! de todos estes inversveis e igual a 1 em Z/pZ.
Vamos ilustar a idea geral da prova com um caso particular primeiro. Por exemplo, para p = 11 podemos
formar parzinhos, cujos produtos sao iguais a 1, isto e, parzinhos da forma {a, a1 }:
{2, 6}

{3, 4}

{5, 9}

{7, 8}

Somente 1 e 10 = 1, que sao seus proprios inversos, ficam isolados. Assim, temos
10! = 1 10 = 1
que era o queramos demonstrar.
No caso geral, observe que somente 1 sao seus proprios inversos pois pela alternativa de primo temos
x2 1

(mod p)

p | x2 1 p | (x 1)(x + 1)
p | x 1 ou p | x + 1 x 1 (mod p)

Assim, podemos agrupar os demais elementos em pares da forma {a, a1 }, de modo que
(p 1)! = 1 (1) = 1
Reciprocamente, suponha agora que p > 1 seja um n
umero que satisfaca (p 1)! 1 (mod p). Ent
ao
todo 1 a p 1 e primo como p, j
a que a e inversvel modulo p: um inverso e dado por (p 1)!/a (que
e um inteiro!). Porem isto implica que p e primo: se p = ab para 1 < a, b < p ent
ao (a, p) = a 6= 1, o que
contradiz o fato anterior.
Corol
ario 51. Se p e um primo tal que p 1 (mod 4) ent
ao a equaca
o quadr
atica
x2 1 (mod p)
admite soluca
o.
Demonstraca
o. Basta combinar o teorema de Wilson acima com o truque do complementar. Agrupando a
com a = p a para a = 1, 2, . . . , p1
2 , obtemos

p 1 2
(p 1)! (1)(p1)/2 1 2 . . .
2

(mod p)

Mas como p 1 (mod 4), temos que (p 1)/2 e par. Logo, pelo teorema de Wilson, a equacao acima e
equivalente a

p 1 2
(mod p)
1 1 2 . . .
2
e assim podemos tomar x = 1 2 . . .

p1
2 .

25

Terminamos esta secao com uma importante


Defini
c
ao 52. O conjunto dos elementos inversveis de Z/mZ
def

(Z/mZ) = {a Z/mZ | (a, m) = 1}


e chamado de grupo dos elementos inversveis de Z/mZ.
1

Note que o produto de dois elementos inversveis a e b e inversvel (o seu inverso e a1 b


(Z/mZ) e fechado com relacao ao produto.

), assim

Exemplo 53. Temos a seguinte tabela de multiplicaca


o de (Z/15Z) :

1 1 2 4 7
2 2 4 8 14
4 4 8 1 13
7 7 14 13 4
8 8 1 2 11
11 11 7 14 2
13 13 11 7 1
14 14 13 11 8


11 13 14

8 11 13 14
1 7 11 13
2 14 7 11
11 2 1 8
4 13 14 7
13 1 8 4
14 8 4 2
7 4 2 1

Em Matem
atica, grupo e o nome emprestado a um conjunto G juntamente com uma operacao binaria
(produto) que satisfaz os seguintes tres axiomas:

1. (Associatividade) Para quaisquer a, b, c G, (a b) c = a (b c)


2. (Existencia de elemento neutro) Existe um elemento e G tal que, para todo a G, a e = e a = a
3. (Existencia de inverso) Para qualquer elemento a G existe um elemento a1 G tal que a a1 =
a1 a = e
Se, alem dos tres axiomas acima, o grupo G satisfaz
4. (Comutatividade) Para quaisquer a, b G, a b = b a
ent
ao G e chamado de grupo abeliano (em homenagem ao matematico noruegues Niels Henrik Abel (1802
1829)).

2.3

Teorema Fundamental da Aritm


etica

Vamos agora provar o Teorema Fundamental da Aritmetica sobre a fatoracao u


nica em primos. Embora
velho conhecido de todos n
os desde os tempos do ensino fundamental, os livros escolares n
ao costumam
incluir uma demonstracao deste fato intuitivamente obvio. Entretanto este teorema e longe de ser trivial:
basta apenas mencionar que, embora o resultado fosse conhecido dos matematicos desde os tempos dos
antigos gregos, a primeira demonstracao correta foi obtida somente 2000 anos mais tarde por Gau!
Para mostrar que este teorema e algo especial do conjunto de todos os inteiros Z, facamos um pequeno
experimento mental. Trocando Z pelo conjunto dos pares 2Z, definamos um primo em 2Z da mesma
forma que em Z: um inteiro par e primo se ele n
ao pode ser escrito como produto de dois pares. Assim,
2, 6, 10, 14, . . . sao primos enquanto que 4 = 2 2, 8 = 2 4, 12 = 2 6, 16 = 4 4, etc. sao compostos em
2Z. Mas agora
60 = 6 10 = 2 30

sao duas decomposicoes distintas de 60 em primos!


Muito bem, sem maiores delongas, eis o celebre

26

Teorema 54 (Teorema Fundamental da Aritmetica). Seja n > 1. Ent


ao n pode ser escrito como produto
de potencias de primos:
n = pe11 pekk

onde pi denotam primos distintos. Alem disso, esta fatoraca


o (dita canonica) e u
nica a menos de uma
permutaca
o dos primos pi .

Demonstraca
o. A demonstracao e pelo PIF, sendo que o teorema e claro para n = 2 (base de inducao):
sendo 2 primo, ele e a sua propria fatoracao canonica, que e u
nica pois em qualquer fatoracao de 2 os fatores
primos devem ser menores ou iguais a 2, logo so h
a uma possibilidade.
Suponha agora que n > 2. Vamos primeiramente mostrar a existencia da fatoracao. Se n e primo, n
ao
h
a o que fazer, logo supomos que n e composto, digamos n = ab com 1 < a, b < n. Temos, pela hipotese de
inducao, que a e b podem ambos serem escritos como produtos de potencias de primos, logo o mesmo vale
para n = ab.
Agora suponha que tenhamos duas fatoracoes (potencialmente distintas)
n = pe11 pekk = q1f1 qlfl

()

onde pi e qi sao primos. Pela alternativa de primo, temos que p1 divide algum dos qi , e renumerando podemos
supor que p1 | q1 . Mas q1 e primo! Assim, a u
nica possibilidade e p1 = q1 , e cancelando um fator p1 = q1
em () obtemos
pe11 1 pekk = pf11 1 q2f2 qlfl

Por hipotese de inducao, estas duas fatoracoes sao a mesma a menos da permutacao de seus fatores. Assim,
podemos supor k = l, pi = qi para todo i, e1 1 = f1 1 e ei = fi para i 2. Mas isto implica que as
fatoracoes () tambem sao iguais, completando a demonstracao.
Podemos agora obter alguns axiomas vistos na escola:
Corol
ario 55. Seja

n = pe11 pekk

a fatoraca
o can
onica do natural n em primos distintos pi . Temos que os divisores naturais de n s
ao exatamente os n
umeros da forma
d = pf11 pfkk
com 0 fi ei para todo i. Assim, o n
umero de divisores positivos de n e igual a
(e1 + 1)(e2 + 1) (ek + 1)

Demonstraca
o. Temos que todo d = pf11 pfkk com 0 fi ei e um divisor de n pois n/d e inteiro.
Reciprocamente, se n = dt, ent
ao escrevendo as fatoracoes canonicas de d, t e n e comparando os dois lados,
obtemos que os primos que dividem d e t sao os mesmos que dividem n e, para um primo fixado pi , os
expoentes fi e gi em d e t devem satisfazer fi + gi = ei , logo 0 fi , gi ei . Para mostrar a formula do
n
umero de divisores, basta notar que cada fi pode ser escolhido independentemente entre ei + 1 possveis
valores 0, 1, . . . , ei .
Corol
ario 56. Sejam

m = pe11 pekk
n = pf11 pfkk

as fatoraco
es can
onicas dos naturais m e n em primos distintos pi com ei , fi 0 (de modo que podemos
assumir a mesma quantidade de primos nas fatoraco
es acima). Temos
Y min{e ,f }
Y max{e ,f }
i i
i i
mdc(m, n) =
e
mmc(m, n) =
pi
pi
1ik

1ik

e portanto

mdc(m, n) mmc(m, n) = m n
27

Demonstraca
o. As f
ormulas para o mdc e mmc decorrem da descricao dos divisores do corol
ario anterior,
enquanto que a u
ltima f
ormula e consequencia do fato de que, para todo i, max{ei , fi } + min{ei , fi } =
ei + fi .
A prop
osito, a u
ltima f
ormula do corol
ario anterior fornece um metodo eficiente para o c
alculo do mmc
de dois n
umeros: calcule primeiro o mdc via algoritmo de Euclides e depois faca mmc(m, n) = mn/(m, n).
Para n
umeros grandes, fatorar em primos e uma operacao muito mais custosa do que a que acabamos de
descrever!
Vejamos algumas aplicacoes dos resultados acima.

Exemplo 57. Seja n um inteiro positivo que n


ao e um quadrado perfeito. Ent
ao n e um n
umero irracional.
Soluca
o. Seja n = pe11 pekk a fatoracao canonica de n em potencias de primos pi distintos. Se todos os
ei s fossem pares, n seria um
e1 .

quadrado perfeito, logo existe um ei que e mpar, digamos


A demonstracao de que n e irracional e por reducao ao absurdo. Suponha que n = ab com a, b inteiros
positivos. Ent
ao
a2 = nb2
()
Pelo teorema fundamental da aritmetica, a potencia de p1 em ambos os lados de () deve ser igual. Se p
ea
1
2
potencia de p1 na fatoracao can
onica de a, ent
ao p2
ser
a
a
pot
e
ncia
de
p
na
fatora
c
a

o
can
o
nica
de
a
,
isto
1
1
e, uma potencia com expoente par. Analogamente, o expoente da potencia de p1 na fatoracao canonica de
b2 tambem e par, digamos 2, e como a potencia de p1 na fatoracao de n e pe11 temos que a potencia de p1
na fatoracao do lado direito de () e 2 + e1 , um n
umero mpar. Logo as potencias de p1 do lado esquerdo
e direito de () n
ao podem ser iguais, uma contradicao.
Exemplo 58. Se (m, n) = 1 e mn e uma k-esima potencia perfeita, ent
ao m e n s
ao ambos k-esimas
potencias perfeitas.
Soluca
o. Temos mn = xk para algum x. Se pt e a potencia do primo p na fatoracao canonica de x, temos
que pkt e a potencia de p na fatoracao canonica de mn. Porem, como (m, n) = 1, temos que p ou divide m
ou divide n, mas n
ao ambos; digamos que p | m. Desta forma pkt e a potencia de p na fatoracao canonica
de m. Como o raciocnio anterior e valido para qualquer primo, temos que m e n sao ambos produtos de
potencias de primos cujos expoentes sao m
ultiplos de k, logo sao ambos k-esimas potencias perfeitas.
Observe que se (m, n) 6= 1 a conclusao n
ao e valida! Por exemplo, temos que nem m = 31 e nem
3
n = 3 sao quadrados perfeitos, mas mn = 34 e um quadrado perfeito! Na situacao acima, quando
(m, n) = 1, isto n
ao ocorre pois n
ao e possvel quebrar a potencia pkt entre m e n.

O fato anterior e um fato extremamente u


til na resolucao de equacoes diofantinas. Veja o
Exemplo 59. Determine todas as soluco
es de
x2 + y 2 = z 2
com x, y, z inteiros positivos dois a dois primos entre si.
Soluca
o. Note que temos uma fatoracao (fatore e fature!)
x2 = z 2 y 2 = (z y)(z + y)
Ser
a que (z y, z + y) = 1? Em caso afirmativo, poderamos utilizar o exemplo acima para concluir que
z y e z + y sao ambos quadrados perfeitos. Seja d = (z y, z + y). Pelo d divide, temos que d divide a
soma 2z e a diferenca 2y destes dois fatores. Porem, como (z, y) = 1 por hipotese, temos que d | 2. Assim
temos que analisar a paridade destes termos.
28

Como x, y, z sao primos entre si, nem todos eles podem ser pares. Logo h
a exatamente dois mpares e um
par. Por outro lado, analisando modulo 4 como no exemplo 34, temos que n
ao podemos ter x e y mpares
e z par, pois neste caso x2 + y 2 2 (mod 4) enquanto que z 2 0 (mod 4). Logo z e mpar e por simetria
podemos assumir que x e mpar e y e par. Logo z y e z + y sao mpares e assim d = 1.
Logo pelo exemplo anterior, temos que existem naturais m e n mpares com m < n tais que x = mn,
z y = m2 e z + y = n2 . Resolvendo o sistema, obtemos
x = mn

y=

n 2 m2
2

z=

n 2 + m2
2

()

S
o falta determinar condicoes para que x, y, z sejam dois a dois primos entre si. Note que qualquer fator
primo comum a dois dentre os inteiros x, y, z deve dividir o terceiro, logo este n
umeros serao dois a dois
primos entre si se, e somente se, (y, z) = 1 (por exemplo). Temos (c.f. demonstracao do algoritmo de
Euclides)
 n 2 m2 n 2 + m2   n 2 m2
n 2 + m2 n 2 + m2 
=
,
+
,
2
2
2
2
2

2
2

n
+
m
()
=
n2 ,
= n2 , n2 + m2 = (n2 , m2 )
2

(y, z) =

onde () utiliza o fato de que n2 e mpar, de modo que multiplicar a segunda entrada por 2 n
ao altera o mdc.
Assim, (y, z) = 1 (m2 , n2 ) = 1 (m, n) = 1 (utilize a expressao do mdc em termos da fatoracao
de m e n).
Efim, a resposta final e dada por () para m > n naturais mpares primos entre si.
Conclumos esta secao com uma
Defini
c
ao 60. Seja p um primo. Dizemos que p divide exatamente m (em smbolos p k m) se e o
expoente de p na fatoraca
o can
onica de m (ou, equivalentemetne, se p e a maior potencia de p que divide
m).
Eis uma aplicacao cl
assica:
Teorema 61 (Fatores do Fatorial). Seja p um primo. Sendo
jnk j n k j n k
+ 2 + 3 +
=
p
p
p
temos que p k n!

 
Observe que a soma acima e finita pois os temos pni sao eventualmente iguais a zero. E relembrando:
para um n
umero real x, x denota a parte inteira t de x, isto e, o u
nico inteiro t tal que x 1 < t x.
 
Demonstraca
o. No produto n! = 1 2 . . . n, apenas os m
ultiplos de p contribuem com um fator p. H
a np
 
tais m
ultiplos entre 1 e n. Destes, os que sao m
ultiplos de p2 contribuem com um fator p extra e h
a pn2
tais fatores. Dentre estes u
ltimos, os que sao m
ultiplos de p3 contribuem com mais um fator p e assim por
diante, resultando na f
ormula acima.
Exemplo 62. Determine com quantos zeros termina 100!
Soluca
o. O problema e equivalente a determinar a maior potencia de 10 que divide 100! Como h
a muito
mais fatores 2 do que 5 em 100!, temos que determinar apenas a maior potencia de 5 que divide 100! e agora
e so aplicar a f
ormula anterior: temos 100/5 + 100/52 = 24, logo a maior potencia de 5 que divide 100!
24
e 5 . Portanto 100! termina em 24 zeros.

29

2.4

Teorema Chin
es dos Restos

Vamos agora considerar o problema de resolver sistemas de congruencias lineares. Vejamos um


Exemplo 63. Determine o menor inteiro positivo x que deixa restos 1 e 13 nas divis
oes euclidianas por
109 e 1001, respectivamente.
Soluca
o. Devemos encontrar a menor solucao positiva do sistema
(
x 1 (mod 109)
x 13 (mod 1001)
Da primeira congruencia temos que x = 1 + 109k para algum k Z. Substituindo na segunda, obtemos
portanto
1 + 109k 13 (mod 1001) 109k 12 (mod 1001)
()
Agora, multiplicando pelo inverso multiplicativo de 109 m
odulo 1001, que e igual a 450, temos
() 450 109k 450 12 (mod 1001) k 395 (mod 1001)
Portanto k = 395 + 1001t e assim
x = 1 + 109 (395 + 1001t)

x = 43056 + 109 1001t

x 43056 (mod 109 1001)

e o menor inteiro positivo satisfazendo estas condicoes e 43056.


Em muitas aplicacoes, n
ao e necessario obter as solucoes explicitamente, apenas saber se elas existem.
Para isto, temos o importante
Teorema 64 (Teorema Chines dos Restos). Sejam m1 , m2 , . . . , mr inteiros positivos que s
ao primos entre
si, dois a dois, e sejam a1 , a2 , . . . , ar inteiros quaisquer. Ent
ao o sistema de conguencias
x
x

a1

a2
..
.
ar

(mod m1 )
(mod m2 )

(mod mr )

admite uma soluca


o x, u
nica m
odulo m = m1 m2 . . . mr .
Apresentamos duas demonstracoes para este teorema, que ilustram tecnicas distintas.
Primeira prova. Considere a funcao diagonal
Z/mZ Z/m1 Z Z/mr Z
a 7 (a, . . . , a)
Primeiramente, precisamos mostrar que esta funcao est
a bem definida, isto e, que escolhendo outro representante de classe b a em Z/mZ temos b = a em cada Z/mi Z (cuidado, as barras denotam reducao
modulo diferentes n
umeros!) Mas isto decorre facilmente do fato de mi | m, pois b = a + km para algum
k Z de modo que b a (mod mi ) para todo i.
O teorema e equivalente a mostrar que esta funcao e sobrejetora (isto e, que existe um x Z/mZ que
atinge qualquer tupla (a1 , . . . , ar ) Z/m1 Z Z/mr Z. Como o n
umero de elementos em ambos os
30

lados e igual a m = m1 m2 . . . mr , basta mostrar que esta funcao e injetora. Se dois elementos a, b Z/mZ
tem mesma imagem, i.e.,
(a, . . . , a) = (b, . . . , b) a b (mod mi ) para todo i
temos que mi | a b para todo i. Mas como os mi sao dois a dois primos entre si, temos (olhando para a
fatoracao em primos) que m | a b, ou seja, a = b em Z/mZ, que era o que queramos mostrar.

m
m
, mj = 1 (note que m
e um
Segunda prova. Observe que como (mi , mj ) = 1 para i 6= j, temos que m
j
j
m
m
inteiro!) enquanto que mj 0 (mod mi ) para i 6= j. Seja bj um inverso multiplicativo de mj modulo mj
m
e mj sao primos entre si). Tome
(que existe pois m
j
x=
M
odulo mi , temos
x0

m
b i ai
mi

m
m
b 1 a1 + +
b r ar
m1
mr

(mod mi ) x0 ai

(mod mi )

Ent
ao x0 e uma solucao do nosso sistema. Agora se x0 e x1 sao duas solucoes temos x0 x1 (mod mi ) para
cada i. Como (mi , mj ) = 1 para i 6= j temos como na primeira solucao que x0 x1 (mod m), mostrando a
unicidade.
Exemplo 65. Mostre que existem 1000 inteiros consecutivos compostos.
Soluca
o. Uma maneira de garantir que os inteiros consecutivos x + 1, x + 2, . . . , x + 1000 sejam compostos e
torna-los m
ultiplos de primos fixos, com o cuidado de escolher x grande o suficiente para que nenhum destes
n
umeros coincida com um destes primos.
Assim, escolha 1000 primos distintos p1 , . . . , p1000 e considere o sistema
x
x

1 (mod p1 )
2 (mod p2 )
..
.

1000 (mod p1000 )

Pelo Teorema Chines dos Restos, o sistema acima possui infinitas solucoes, logo basta tomar uma solucao x
suficientemente grande.

2.5
2.5.1

Exerccios
B
asicos

24. Resolva as seguintes equaco


es diofantinas lineares:
(a) 172x + 13y = 1
(c) 123x + 130y = 13
(e) 391x + 1377y = 34

(b) 233x + 144y = 1


(d) 280x + 49y = 2
(f ) 22x + 121y = 11

25. Mostre que se x e y s


ao tais que N = (x + 6y)(2x + 5y)(3x + 4y) e divisvel por 7 ent
ao N e m
ultiplo
de 343.
26. Encontre o inverso multiplicativo (ou mostre que ele n
ao existe):
(a) 7 mod 13
(b) 12 mod 143
(c) 22 mod 121
(d) 19 mod 200
31

27. Com quantos zeros termina 2008! quando escrito na notaca


o decimal?
28. Prove o seguinte criterio de divisibilidade por 7: dado um inteiro n, seja d o u
ltimo dgito de n e m o
n
umero obtido a partir de n apagando-se o u
ltimo dgito d; ent
ao n e divisvel por 7 se, e somente se, m 2d
e divisvel por 7. Por exemplo, para n = 8638, temos 863 2 8 = 847 e 84 2 7 = 70, que e divisvel por
7, logo 8638 e divisvel por 7 tambem.
29. Sejam m0 , m1 , . . . , mr inteiros positivos que s
ao dois a dois primos entre si. Mostre que existem r + 1
inteiros consecutivos s, s + 1, . . . , s + r tal que mi | s + i para i = 0, 1, . . . , r.
30. Existem 2009 inteiros consecutivos tal que cada e divisvel por uma centesima potencia de um primo?

2.5.2

Lic
ao de Casa

31. (Est
onia) Determine todos os restos possveis da divis
ao do quadrado de um n
umero primo com 120 por
120.
32. Seja n um n
umero natural e seja

n = pe11 pekk

a fatoraca
o can
onica de n em potencias de primos pi distintos. Mostre que a soma dos divisores positivos
de n e igual a
pek +1 1
pe11 +1 1 pe22 +1 1

... k
p1 1
p2 1
pk 1

2.5.3

Quest
oes de Prova

33. Um ponto inteiro (x, y) Z2 e visvel se (x, y) = 1. Existe um ponto (a, b) Z2 cuja dist
ancia a todo
ponto visvel e pelo menos 1000?
34. (IMO) Encontre um par de inteiros positivos a e b tais que
(i) ab(a + b) n
ao e divisvel por 7;
(ii) (a + b)7 a7 b7 e divisvel por 77 .
35. Seja p(x) um polin
omio n
ao constante com coeficientes inteiros e seja k um inteiro qualquer. Prove que
existe um inteiro m tal que p(m) tem pelo menos k fatores primos distintos.

32

Captulo 3

Teorema de Euler-Fermat
Conforme vimos, para calcular uma potencia ad mod m, e interessante obter algum expoente n tal que
an 1 (mod m) (nossa velha filosofia 1 lembra-se?). O teorema de Euler-Fermat mostra como
encontrar este expoente magico n > 0 de modo que an 1 (mod m).
Primeiramente, vejamos os casos em que podemos esperar a existencia de tal expoente magico. Se
d = (a, m) 6= 1, tal expoente n
ao pode existir, pois caso contrario teramos
an 1 (mod m) an = mx + 1 para algum x
Porem, d divide a e m, logo d divide 1 = an mx, uma contradicao.
No caso em que a e m sao primos entre si, a sim podemos esperar encontrar tal expoente magico. De
fato, em Z/mZ considere as potencias de a
1, a, a2 , a3 , a4 , . . .
Como Z/mZ e um conjunto finito, esta lista contem (muitos) elementos repetidos. Se i > j sao tais que
ai = aj , como a e m sao primos entre si, temos que a e inversvel em Z/mZ (ou seja, a (Z/mZ) ) e desta
forma
ai = aj aij = 1 aij 1 (mod m)
o que mostra a existencia do expoente magico n = i j > 0.
para isto que temos o teorema de Euler-Fermat.
Melhor do que saber que ele existe, e saber calcula-lo. E

3.1

Fun
c
ao de Euler

Defini
c
ao 66. Seja n um inteiro positivo. A funca
o de Euler e definida como
(m)

def

quantidade de inteiros 1 d m tais que d e m s


ao primos entre si
n
umero de elementos do grupo (Z/mZ)

A funcao de Euler foi introduzida, advinhem, pelo grande matematico suco Leonhard Paul Euler
(17071783), em conex
ao com o teorema de Euler-Fermat que discutiremos a seguir. Esta funcao tambem
e conhecida em Ingles por Euler totient function, nome dado por James Joseph Sylvester (18141897),
matematico britancio que adorava inventar novas palavras complicadas.
Exemplo 67. Eis alguns exemplos:
(1) = 1;
33

(12) = 4 pois (Z/12Z) = {1, 3, 5, 7} tem 4 elementos;


temos que

p e um n
umero primo (p) = p 1

De fato, se p e primo, ent


ao dentre os n
umeros 1, 2, . . . , p, todos s
ao primos com p com exceca
o do
pr
oprio p. Por outro lado, se p = ab e composto com 1 < a, b < p, ent
ao na lista 1, 2, . . . , p, os n
umeros
a, b e p n
ao s
ao primos com p, logo (p) < p 1 neste caso. Finalmente, se p n
ao e primo e nem
composto (ent
ao o que ele e?), temos que p = 1 (ah, e verdade, 1 n
ao e nem primo nem composto!)
logo (p) = 1 6= p 1 = 0.
Calcular diretamente atraves da definicao pode n
ao ser muito pratico, ent
ao vamos provar
Lema 68 (Calculando ). Seja

k
2
1
m = p
1 p2 . . . pk

a fatoraca
o can
onica de m em potencias de primos distintos pi . Ent
ao
(m)

=
=

k 1
1 1
1
2 1
k
(p
) (p2 p
) . . . (p
)
1 p1
2
k pk
 2




1
1
1
1
... 1
m 1
p1
p2
pk

Demonstraca
o. Seja
Q

def

probabilidade de que um n
umero 1 d m seja primo com m
(m)
m

Vamos calcular esta probabilidade de outra maneira. Temos que d e primo com m se, e somente se, d n
ao e
a
m
ultiplo de nenhum fator primo pi que divide m. A probabilidade de que d seja m
ultiplo de pi e p1i ja que h
pi possveis restos igualmente provaveis para d mod pi . Assim, a probabilidade de que d n
ao seja m
ultiplo
ao seja m
ultiplo de nenhum pi e
de pi e 1 p1i . Portanto a probabilidade Q de que d n

 



1
1
1
Q= 1
1
... 1
p1
p2
pk
Comparando as duas f
ormulas para Q, obtemos o resultado.
Exemplo 69. Pela f
ormula temos



1
1
(12) = 12 1
1
=4
2
3
que, incrivelmente, coincide com o valor que encotramos anteriormente!
Utilizando a f
ormula acima, e f
acil provar o corol
ario a seguir, que deixamos como exerccio para voce:
Corol
ario 70. A funca
o e multiplicativa, isto e, para (m, n) = 1 temos
(m n) = (m) (n)


Esta relaca
o s
o vale quando m e n s
ao primos entre si!

3.2

Teorema de Euler-Fermat

Agora estamos prontos para o teorema propriamente dito! Comecamos enunciando o teorema e suas variantes.
Em seguida, esbocamos a sua prova num caso particular, mas que ja encerra todos os ingredientes principais
da demonstracao no caso geral, que e atacado logo em seguida.
34

3.2.1

Enunciado do teorema e suas variantes

Teorema 71 (Euler-Fermat). Se (a, m) = 1, ent


ao
a(m) 1
ao
Equivalentemente, se a (Z/mZ) ent

(mod m)

a(m) = 1

Trocando em mi
udos: se a base a e prima com o modulo m, ent
ao existe um expoente, a saber (m), tal
que a(m) 1 (mod m). Note que uma vez encontrado um tal expoente magico, h
a infinitos deles: para
qualquer k 0 temos automaticamente
ak(m) 1 (mod m)
Exemplo 72. (Para os ceticos) Tome a = 3 e m = 14. Temos (14) = 6. Fazendo as contas na raca:
33 1 (mod 14) = 36 1 (mod 14)
Incrvel! N
ao e que funciona mesmo. . .
O teorema, no formato acima, foi demonstrado por Euler em 1736. Um caso particular ja havia sido
(de)monstrado pelo grande matematico frances Pierre de Fermat (16011665) quase um seculo antes, a saber
o caso em que m = p e um primo. Neste caso, temos (p) = p 1, logo
ap1 1

(mod p)

para todo a primo com p (isto e so um jeito complicado de dizer que a n


ao e um m
ultiplo de p). Agora
vamos multiplicar ambos os membros da congruencia por a. Temos
ap a

(mod p)

()

Pensemos mais um pouco. Temos que () so poderia falhar se a n


ao fosse primo com p, isto e, so se
p | a, o que na linguagem das congruencias e o mesmo que a 0 (mod p). Ora, substituindo este valor em
(), obtemos uma relacao verdadeira ainda. Assim () vale para todo inteiro a. Obtemos como corol
ario o
resultado original de Fermat:
Teorema 73 (Pequeno Teorema de Fermat). Seja p um primo. Ent
ao para todo inteiro a temos
ap a

(mod p)

O teorema acima admite uma prova combinatoria bem simples e interessante. Suponha que desejamos
formar colares com p contas e temos contas de a cores distintas. De quantas maneiras podemos montar estes
colares, sendo que dois colares sao considerados o mesmo se um pode ser obtido a partir do outro por meio
de uma rotacao?
Se n
ao houvesse rotacoes envolvidas, a resposta seria apenas o n
umero de sequencias (a1 , a2 , . . . , ap )
onde ai representam uma dentre a cores, ou seja, o n
umero de colares seria ap . Com as rotacoes, cada colar
distinto pode corresponder a mais de uma sequencia (a1 , a2 , . . . , ap ) diferente. Temos dois casos a analisar:
Todas as contas tem a mesma cor. Neste caso h
a a colares.
O colar n
ao e monocrom
atico. Neste caso, cada colar d
a origem, por rotacao, a exatamente p sequencias
(a1 , a2 , . . . , ap )
(a2 , a3 , . . . , ap , a1 )
(a3 , a4 , . . . , ap , a1 , a2 )
..
.
(ap , a1 , a2 , . . . , ap1 )
35

De fato, vamos mostrar que se duas das sequencias acima sao iguais ent
ao o colar e monocrom
atico.
Suponha por exemplo que (a1 , a2 , . . . , ap ) = (ai+1 , ai+2 , . . . , ai+p ) para algum 0 < i < p (onde os
ndices sao tomados m
odulo p), isto e, a sequencia (a1 , a2 , . . . , ap ) e igual ao seu i-esimo shift.
Ent
ao teramos
shift
shift
shift
shift
shift
a1 = ai+1 = a2i+1 = a3i+1 = a4i+1 =
(onde novamente os ndices sao tomados m
odulo p). Note que basta mostrar que os n
umeros 1, i +
1, 2i + 1, 3i + 1, . . . , (p 1)i + 1 percorrem todos os inteiros modulo p (ou seja, que eles formam um
sistema completo de resduos modulo p pela definicao 35). Se houvesse uma repeticao ri + 1 si + 1
(mod p), como i e inversvel modulo p (0 < i < p), teramos r s (mod p). Mas como 0 r, s < p,
isto implica r = s, ou seja, a repeticao ocorreu porque comecamos como o mesmo elemento! Logo os
p n
umeros 1, i + 1, 2i + 1, 3i + 1, . . . , (p 1)i + 1 realmente formam um sistema completo de restos,
terminando a prova.
O total de colares e portanto a soma dos monocrom
aticos e dos policromaticos, isto e,
a+

ap a
p
p

Mas este total e um n


umero inteiro (afinal de contas, estamos contando n
umero de colares)! Logo a pa e
p
um n
umero inteiro, isto e, a a (mod p), and thats Fermat for you!
O teorema de Fermat tem uma consequencia fant
astica, o sonho de todo aluno que estuda aritmetica na
escola!
Corol
ario 74 (O sonho de todo estudante). Se p e um primo ent
ao
(a + b)p ap + bp

(mod p)

para quaisquer inteiros a e b.


Demonstraca
o. De fato, aplicando o pequeno teorema de Fermat tres vezes temos

(a + b) a + b (mod p)
p
a a (mod p)

p
b b (mod p)

Assim, modulo p, (a + b)p a + b ap + bp .

3.2.2

Um esboco da demonstrac
ao

Vamos ver um caso particular: facamos a = 3 e m = 14. Vamos mostrar que 3(14) 1 (mod 14). Bom,
precisamos conhecer (14). Para isso, listemos os n
umeros positivos menores que 14 e primos com 14:
1

3 5

11 13

Vamos agora multiplicar todos estes n


umeros por a = 3 e calcular cada um deles modulo 14:
13
33
53
93
11 3
13 3

3
9
1
13
5
11
36

(mod
(mod
(mod
(mod
(mod
(mod

14)
14)
14)
14)
14)
14)

Vejam! Todos os n
umeros positivos menores que 14 e primos com 14 apareceram novamente, so que em
outra ordem! Multiplicando todas estas (14) congruencias, temos
(1 3 5 9 11 13) 3(14) (3 9 1 13 15 11) (mod 14)
Como 1 3 5 9 11 13 e primo com 14 (estamos multiplicando n
umeros sem fatores em comum com 14,
logo o produto continua sem fatores em comum com 14), este produto e inversvel modulo 14 e portanto
cancelando obtemos
3(14) 1 (mod 14)
Lembra algo? Se voce disse Euler-Fermat, acertou!

3.2.3

No caso geral. . .

O que fizemos? Primeiro, listamos os n


umeros positivos a1 , a2 , . . . , a(m) menores que m e primos com
m. A multiplicamos cada um deles por a e, calculando tudo modulo m, obtivemos os mesmo n
umeros
a1 , a2 , . . . , a(m) em outra ordem. Depois era so multiplicar tudo.
Ser
a que este fenomeno e uma mera coincidencia? Obra do acaso? Ou seria consequencia do
Teorema 75 (Gira-Gira). Seja a primo com m. Multiplicaca
o por a permuta os elementos de (Z/mZ)
e de Z/mZ. Explicitamente: se
T = {a1 , a2 , . . . , a(m) }

e o conjunto dos n
umeros positivos menores ou iguais a m e primos com m (de modo que T mod m =
(Z/mZ) ), ent
ao
T = {a a1 mod m, a a2 mod m, . . . , a a(m) mod m}
Se

S = {b1 , b2 , . . . , bm }

e um sistema completo de resduos m


odulo m (de modo que S mod m = Z/mZ) ent
ao
{a b1 , a b2 , . . . , a bm }
tambem e um sistema completo de resduos m
odulo m.
Demonstraca
o. Vamos demonstrar que multiplicacao por a permuta os elementos de (Z/mZ) e deixamos
o outro caso para o leitor.
Sejam
a1 , a2 , . . . , a(m)
os (m) elementos de (Z/mZ) . Temos que
a a1 , a a2 , . . . , a a(m)
tambem sao elementos de (Z/mZ) pois a (Z/mZ) por hipotese e (Z/mZ) e fechado por multiplicacao.
Para mostrar que esta lista contem todos os elementos de (Z/mZ) (em alguma ordem), basta mostrar que
eles sao todos distintos, j
a que h
a (m) deles.
Mas isto e f
acil: se houvesse uma repeticao a ai = a aj para dois elementos distintos ai e aj , cancelando
a obteramos ai = aj , um absurdo.
Vamos agora concluir a demonstracao do teorema de Euler-Fermat. Com a notacao acima, pelo giragira, temos que
a a1
a a2

..
.

a1
a2

a a(m)

a(m)

37

(mod m)
(mod m)

(mod m)

onde os ai sao uma permutacao dos ai . Assim, multiplicando estas equacoes, obtemos
Y
Y
ai (mod m)
ai
a(m)
1i(m)

1i(m)

Como cada ai e inversvel modulo m, o produto 1i(m) ai tambem e inversvel modulo m. Pelo cancelamento, temos que a(m) 1 (mod m), como desejado.

3.3

Utilizando o teorema de Euler-Fermat

Mostraremos aqui algumas aplicacoes do teorema de Euler-Fermat. A primeira e facilitar nossas contas.
Exemplo 76. Calcule o resto da divis
ao de 22001 por 101.
Soluca
o. Temos que 2 e 101 sao primos entre si. Utilizando o teorema de Euler-Fermat, temos (101) = 100
e portanto, como 2001 = 100 20 + 1,
22001 = 2100

20

2 120 2

(mod 101) = 22001 2

(mod 101)

Observe que podemos pensar isto da seguinte forma: como 2001 1 (mod (101)), temos 22001 21
(mod 101). Ou seja, as potencias de 2 modulo 101 sao peri
odicas modulo (101).
Eis uma aplicacao n
ao exatamente do teorema de Euler-Fermat mas sim do gira-gira:
Exemplo 77. Mostre que
S = 16 + 26 + 36 + 46 + + 1006
e divisvel por 101.
Soluca
o. Multiplicando S por 26 temos
26 S = (2 1)6 + (2 2)6 + + (2 100)6
Pelo gira-gira, temos que {2 1, 2 2, . . . , 2 100} = {1, 2, . . . , 100}, assim
26 S 16 + 26 + 36 + 46 + + 1006

(mod 101)

26 S S

(mod 101)

(26 1)S 0 (mod 101)


S 0 (mod 101)

ja que 26 1 = 63 e primo com 101.


Outra aplicacao usa simplesmente a existencia do expoente magico (m).
Exemplo 78. Mostre que existe uma potencia de 3 cuja representaca
o decimal contem pelo menos 1000
zeros sucessivos.
Demonstraca
o. Como (3, 101001 ) = 1, podemos escrever
1001

3(10
1001

Assim, os u
ltimos 1001 dgitos de 3(10

(mod 101001 )

sao 0000 . . . 1 (1000 zeros), o que termina o problema.

Exemplo 79. Mostre que existem infinitos n


umeros da forma 200 . . . 01 que s
ao m
ultiplos de 2001.

38

Soluca
o. Seja n o n
umero de zeros em 200 . . . 01. Temos 200 . . . 01 = 2 100 . . . 00 + 1 = 2 10n+1 + 1. Assim,
basta encontrarmos infinitos valores de n > 2 para os quais 2 10n+1 + 1 0 (mod 2001). Mas
2 10n+1 + 1 0 (mod 2001)

2 10n+1 + 1 2001 (mod 2001)

2 10n+1 2000 (mod 2001)

10n+1 103

(mod 2001)

Como n + 1 > 3, temos ent


ao que encontrar infinitos ns tais que
10n2 1

(mod 2001)

()

Observe que a expressao acima e parecida com a do teorema de Euler-Fermat. De fato, como (10, 2001) =
1, temos, pelo teorema de Euler-Fermat, que existe um expoente (2001) tal que
10(2001) 1

(mod 2001)

Mas a partir de um expoente, e f


acil encontrar infinitos: elevando a congruencia anterior a um n
umero
positivo k, temos
10(2001)k 1 (mod 2001)
()
Assim, tomando n 2 = (2001)k n = (2001)k + 2, temos, a partir de (), o que queremos
em (). Assim, existem infinitos n
umeros da forma 200 . . . 01 (aqueles com n = (2001)k + 2 zeros) que sao
m
ultiplos de 2001.
O proximo problema mostra como o teorema de Euler-Fermat e u
til para reconhecer quando n
ao h
a
solucoes para certas equacoes modulo m.
Exemplo 80. Mostre que se m e n s
ao inteiros tais que 103 divide m2 + n2 ent
ao 103 divide m e n.
ao m 6= 0 tambem: caso contrario, como m2 + n2 0
Soluca
o. Suponha por absurdo que n 6= 0. Ent
(mod 103), teramos que m 0 (mod 103) = n2 0 (mod 103) 103 | n2 103 | n n 0
(mod 103) onde a equivalencia 103 | n2 103 | n e valida pois 103 e primo.
Como n 6= 0 e 103 e primo, temos que n e inversvel em Z/103Z. Multiplicando a hipotese inicial
m2 + n2 = 0 por (n1 )2 obtemos
m2 + n2 = 0 = (m n1 )2 + 1 = 0 (m n1 )2 = 1,
isto e, 1 e um quadrado perfeito modulo 103. Mas isto e impossvel: elevando ambos os lados a (103
1)/2 = 51, obtemos
(m n1 )2 = 1 = (m n1 )102 = 1

o que contradiz o teorema de Euler-Fermat (m n1 )102 = 1 (como m 6= 0, temos m n1 6= 0, logo podemos


aplicar o teorema; por outro lado 1 6 1 (mod 103)).
O proximo exemplo ilustra a utilizacao do binomio de Newton juntamente com o teorema de EulerFermat.
Exemplo 81. Dizemos que um n
umero natural n tem a propriedade P se, e somente se,
n | an 1 = n2 | an 1
para a Z.
(a) Mostre que todo n
umero primo possui a propriedade P .
(b) Mostre que existem infinitos n
umeros compostos que possuem a propriedade P .
39

Soluca
o. Podemos escrever a propriedade P como
an 1 (mod n) = an 1

(mod n2 )

(a) Suponha que n seja primo. Temos an a (mod n) pelo pequeno teorema de Fermat, logo
an 1 (mod n) = a 1 (mod n) a = kn + 1 para algum k inteiro
Devemos mostrar que (kn + 1)n 1 (mod n2 ) para n primo. Pela formula do binomio de Newton,
(kn + 1)n =

 
 
 
n  
X
n
n
n
n
(kn)i 1ni =
(kn)n + +
(kn)2 +
kn + 1
i
n
2
1
i=0


Tomando (kn + 1)n mod n2 , temos que todos as parcelas da forma ni (kn)i para i 2 sao divisveis por
n2 . Logo
 
n
(kn + 1)n
(kn) + 1 = n kn + 1 1 (mod n2 ),
1
que e o que queramos demonstrar.
(b) Vamos encontrar infinitos n
umeros compostos que possuem a propriedade P . Os n
umeros primos parecem
ser otimos exemplos, ent
ao podemos tentar n = 2p onde p e primo mpar.
Novamente pelo pequeno teorema de Fermat ap a (mod p) e desta forma a2p a2 (mod p). Portanto
an 1

(mod n)

a2p 1 (mod 2p) a2 1 (mod p)


a2 = kp + 1 para algum k inteiro

Alem disso, como n e par, de an 1 (mod n) temos tambem que a deve ser mpar.
Vamos mostrar agora que n = 2p tem a propriedade P . Devemos mostrar que a2p = (kp + 1)p 1
(mod 4p2 ) para p > 2 primo. Isso e o mesmo que mostrar que: (i) (kp + 1)p 1 (mod 4); e (ii) (kp + 1)p 1
(mod p2 ).
2
Ja demonstramos (ii) no item (a). Mostremos (i). Lembrando que (kp+1)p = a2p = (ap ) e que a e mpar
p
p
2p
(e consequentemente a tambem), temos a = 2 + 1 para algum inteiro e assim a = 42 + 4 + 1 1
(mod 4), o que completa nossa demonstracao.

3.4
3.4.1

Exerccios
B
asicos

36. Mostre que 22225555 + 55552222 e divisvel por 7.


37. Calcule os dois u
ltimos dgitos de 72001 .
38. Mostre que
S = 148 + 248 + 348 + + 9648
e divisvel por 97.
39. Mostre que existem infinitos n
umeros da forma 19999 . . . 9991 que s
ao m
ultiplos de 1991.
40. Mostre que se m e n s
ao inteiros tais que 1999 divide m2 + n2 ent
ao 1999 divide m e n.

40

3.4.2

Lic
ao de casa

41. Mostre que


(a) n
ao existe x inteiro tal que 43 | x2 + 1
(b) n
ao existe x inteiro tal que 103 | x3 2
(c) se 43 | x2 + y 2 ent
ao 43 | x e 43 | y
(d) se 103 | x3 2y 3 ent
ao 103 | x e 103 | y
42.

(a) Mostre que se p = 4k + 3 (k Z) e primo e m e n s


ao inteiros tais que p divide m2 + n2 , ent
ao p
divide m e n.

(b) Mostre que se t = m2 + n2 pode ser escrito como soma de dois quadrados perfeitos, ent
ao todos os
fatores primos da forma 4k + 3, k inteiro, devem aparecer na fatoraca
o de t com expoente par.
43. Mostre que para todo inteiro positivo n existe
(a) uma potencia de 7 cuja representaca
o decimal contem pelo menos n zeros sucessivos.
(b) uma potencia de 2 cuja representaca
o decimal contem pelo menos n zeros sucessivos.
Dica: no item (b), aplique o teorema de Euler-Fermat para a = 2 m
odulo 5k .
44. Seja p um primo.
(a) Mostre que para 0 < k < p o coeficiente binomial

p
k

e divisvel por p.

(b) Utilize o item anterior e o bin


omio de Newton para provar, por induca
o em a, que ap a (mod p)
(pequeno Teorema de Fermat) para todo natural a.
45. Mostre que existem infinitos n
umeros da forma 100000 . . . 0001 que s
ao divisveisl por 49.

3.4.3

Quest
oes de Provas

46. (IMO) Sejam a e b inteiros positivos tais que 15a+ 16b e 16a 15b sejam quadrados perfeitos. Encontrar
o menor valor que pode assumir o menor destes quadrados.
Dica: considere (15a + 16b)2 + (16a 15b)2 e analise m
odulo um n
umero conveniente.
47. Um n
umero inteiro positivo e denominado n
umero duplo se sua representaca
o decimal consiste de um
bloco de dgitos n
ao iniciado por zero seguido imediatamente de um bloco identico. Por exemplo, 360360 e um
n
umero duplo, mas 36036 n
ao e. Mostre que existem infinitos n
umeros duplos que s
ao quadrados perfeitos.
Dica: tente encontrar n
umeros duplos com 6, 8 ou 10 dgitos!
48. Prove que o conjunto {2k 3 | k N} contem um subconjunto infinito cujos membros s
ao primos dois
a dois.

41

Captulo 4

Ordem e Razes Primitivas


Neste captulo, estudaremos propriedades relacionadas ao grupo (Z/mZ) . O conceito fundamental e o de
ordem de um inteiro a modulo m, que e uma especie de perodo para as potencias de a modulo m. Em
seguida, estudaremos os inteiros a que possuem perodo maximo (m), as chamadas razes primitivas.

4.1

Ordem e menor divide

Defini
c
ao 82. Sejam a e m inteiros primos entre si. A ordem de a m
odulo m (ou de a (Z/mZ) ),
denotada por ordm a, e o menor inteiro d > 0 tal que
ad 1 (mod m) ad = 1
A ordem d = ordm a e uma especie de perodo para as potencias de a modulo m, pois como ad = 1,
temos que ai+d = ai e assim as potencias ai , i = 0, 1, 2, . . . se repetem de d em d. Este perodo e o perodo
mnimo pois tomamos d o menor possvel.
Exemplo 83. Para 2 Z/31Z, temos
0

2
1
2
2
2
3
2
4
2

= 1
= 2
= 4
= 8
= 16

2
6
2
7
2
8
2
9
2

=
=
=
=
=

1
2
4
8
16

...

de modo que ord31 2 = 5.


Assim como o perodo mnimo de uma funcao trigonometrica divide qualquer outro perodo desta funcao,
temos aqui tambem o
Teorema 84 (Menor divide). Se at 1 (mod m) ent
ao ordm a | t. Em particular, ordm a divide (m).
Demonstraca
o. Seja d = ordm a e t = d q + r a divisao euclidiana de t por d, onde 0 r < d. Assim, como
ad 1 (mod m), temos
q
at 1 (mod m) ad ar 1 (mod m) ar 1 (mod m)
Se r > 0, temos um expoente menor que d tal que ar 1 (mod m), absurdo. Logo r = 0 e portanto d divide
t. Por outro lado, como a(m) 1 (mod m) por Euler-Fermat, devemos ter d | (m) pelo que acabamos de
demonstrar.
42

Podemos utilizar o menor divide para facilitar o c


alculo da ordem, evitando ter que listar todas as
potencias.
Exemplo 85. Determine ord100 3.
Soluca
o. Para otimizar as contas, vamos quebrar o modulo 100 em potencias de primos. Note que
3(25) 1 (mod 25) e 3(4) 1 (mod 4). Temos (25) = 20 e (4) = 2, portanto temos que 320 1
(mod 25) e 320 1 (mod 4). Como 4 e 25 sao primos entre si e 320 1 e divisvel tanto por 4 como por 25,
temos que 320 1 (mod 100), logo ord100 3 | 20 pelo menor divide.
Para mostrar que ord100 3 = 20, basta testar os divisores maximais de 20, que sao 20/2 = 10 e 20/5 = 4
(qualquer divisor proprio de 20 divide um dos dois n
umeros). Fazendo as contas, temos que 310 49
4
(mod 100) e 3 81 (mod 100). Se ord100 3 fosse algum divisor proprio de 20, uma destas congruencias
deveria ser igual a 1 modulo 100, o que n
ao ocorre. Logo ord100 3 = 20.
O conceito de ordem aparece em diversos problemas em Teoria dos N
umeros. Vejamos dois exemplos
tpicos.
Exemplo 86. Sejam a e n dois inteiros positivos primos entre si. Mostre que n | (an 1).
Soluca
o. Note que n e o menor inteiro positivo tal que an 1 (mod an 1). Assim, n e a ordem de a
modulo an 1 e o resultado segue diretamente do menor divide.
Exemplo 87. Mostre que se n e um inteiro maior que 1, ent
ao n n
ao divide 2n 1.
Demonstraca
o. Suponhamos por absurdo que exista um valor de n > 1 tal que n divide 2n 1. Seja p o
menor primo que divide n. Temos que, como n divide 2n 1, ent
ao p divide 2n 1, isto e, 2n 1 (mod p).
Seja d = ordp 2.
Como (p) = p 1, temos pelo menor divide que

d
(

1 (mod p)
2
d|p1
2p1 1 (mod p) =

d
|n
n
2
1 (mod p)

Logo d e divisor comum de p 1 e n e portanto divide (p 1, n). Mas todos os divisores primos de p 1 sao
menores que p e portanto n
ao aparecem na fatoracao de n pela minimalidade de p. Logo (p 1, n) = 1 e o
u
nico valor possvel para d e 1.
Logo 21 1 (mod p) p | 1, absurdo. Assim, n
ao existe n > 1 tal que n divide 2n 1.

4.2

Lema de Hensel

O lema de Hensel e um resultado que nos permite calcular a potencia exata de um primo que divide um certo
n
umero. O lema em si e apenas uma simples aplicacao do binomio de Newton, mas ele e uma ferramenta
muito u
til na resolucao de equacoes diofantinas, especialmente as que envolvem variaveis no expoente.
Relembrando: para um primo p, escrevemos p k m se p e a maior potencia de p que divide m (ver
definicao 60).
Teorema 88 (Lema de Hensel). Seja p um primo mpar, a um inteiro e n um inteiro positivo. Sejam e
inteiros n
ao negativos, com > 0.
(i) Se p k n e p k a 1 ent
ao p+ k an 1 (atenca
o, p deve dividir a 1 pois > 0! Mas note que p
n
ao precisa dividir n)
(ii) Se n e mpar, p k n e p k a + 1 ent
ao p+ k an + 1 (mesma resalva do item (i)).
43

Demonstraca
o. Vamos demonstrar o item (i) e deixar o item (ii) como exerccio. Observe que n = p k,
sendo que k n
ao e m
ultiplo de p. Primeiro provaremos o resultado para k = 1, ou seja, n = p .
A demonstracao e por inducao sobre . Para = 0 o resultado e obvio. Suponha que o resultado e
t
t
valido para = t, ou seja, que ap 1 = p+t m, com m n
ao divisvel por p. Assim, ap = p+t m + 1.
Elevando os dois lados a p e utilizando binomio de Newton, obtemos
t+1

ap

= (p+t m + 1)p
 
 
 
 
p p
p p1 +t
p p2
p
+t
2
=
1 +
1
p m+
1
(p m) + +
(p+t m)p
0
1
2
p
t+1

ap

1 = p+t+1 (m + p v),

em que v e um inteiro. Logo, como m n


ao e divisvel por p, temos que m + p v tambem n
ao e m
ultiplo de
p, o que conclui a demonstracao para o caso especial k = 1.
Se k > 1, basta observar que

ap

1 = ap

k

1 = (ap 1) (ap

(k1)

+ ap

(k2)

+ + ap + 1)

()

Como ap 1 (mod p) (pois p | a 1 por hipotese) temos que

ap

(k1)

+ ap

(k2)

+ + ap + 1 1 + 1 + + 1 k
{z
}
|

(mod p)

k uns

e como k 6 0 (mod p), temos que o segundo fator em () n


ao e divisvel por p. Logo apenas o primeiro fator
em () contribui com fatores p e pelo caso especial ja provado temos que p+ k an 1.
Exemplo 89. Encontre todos os inteiros n
ao negativos x e y tais que
7y 2 3x = 1
Demonstraca
o. Temos 2 3x = 7y 1. Note que a maior potencia de 3 que divide 7 1 e 3. Seja 3m a maior
potencia de 3 que divide y. Ent
ao, pelo lema de Hensel, a maior potencia de 3 que divide 7y 1 e 3m+1 .
Logo x = m + 1.
Agora vamos utilizar desigualdades. Como 3m divide y, 3m y, e assim
m

2 3m+1 = 2 3x = 7y 1 73 1.
o que impoe uma forte restricao sobre os possveis valores de m, pois para m suficientemente grande o lado
da direita e muito maior que o da esquerda, mas a desigualdade est
a no sentido oposto!
Sendo t = 3m , temos 6t 7t 1 7t  1 + 6t,
que

e
verdadeiro
para t = 0 e t = 1 mas falso

para t > 1, pois nesse caso 7t = (6 + 1)t > 0t 1t + 1t 1t1 6 = 1 + 6t. Assim, a u
nica possibilidade e
t = 1 m = 0, logo x = m + 1 = 1 e assim y = 1.

4.3

Razes Primitivas

possvel que ordm a = (m)? E se isso acontecer, sera que isso e u


Ja sabemos que ordm a | (m). E
til? Este
fato e t
ao u
til que n
umeros com essa propriedade ganham um nome especial:
Defini
c
ao 90. Dizemos que g e raiz primitiva de m quando ordm g = (m).
Um fato simples mas importante e que se g e uma raiz primitiva modulo m ent
ao as potencias de g
geram todo o grupo (Z/mZ) (da a nossa escolha da letra g). Precisamente, temos

44

Lema 91. Se g e raiz primitiva de m ent


ao
(Z/mZ) = {1, g, g 2 , . . . , g(m)1 }
Ou seja, se (a, m) = 1 ent
ao existe i tal que a g i (mod m).
Demonstraca
o. Como (Z/mZ) tem (m) elementos, basta mostrar que dentre as (m) potencias 1, g, g 2 , . . . , g (m)1
ao gji = 1 com
de g n
ao h
a duas repetidas modulo m. Mas se gi = gj para 0 i < j < (m) ent
0 < j i < (m), contradizendo o fato de que g tem ordem (m).
Parece promissor, n
ao? Pena que nem todos os n
umeros admitem razes primitivas. O seguinte teorema
e o principal resultado deste captulo:
Teorema 92 (Razes Primitivas). Os n
umeros que admitem razes primitivas s
ao 2, 4, pn e 2pn , sendo p
primo mpar.
A demonstracao deste teorema e longa e sera dada nas proximas secoes. Vejamos antes alguns exemplos.
Exemplo 93. Seja p um primo. Mostre que
(
0
1 + 2 + + (p 1) mod p =
p1
k

se (p 1) k
se (p 1) | k

Soluca
o. Se (p 1) | k, temos que cada termo da soma acima e congruente a 1 modulo p e o resultado
segue. Suponha agora que (p 1) k e seja g uma raiz primitiva modulo p. Temos portanto
1k + 2k + + (p 1)k 1 + g k + g 2k + + g (p2)k

(mod p)

Gostaramos de somar a PG modulo p acima. Podemos utilizar o mesmo truque da soma da PG usual:
sendo S = 1 + g k + g 2k + + g (p2)k , multiplicando pela razao g k e observando que g (p1)k 1 (mod p)
temos

g k S g k + g 2k + + g (p1)k
k

2k

(mod p)

(p2)k

g S 1 + g + g + + g
(mod p)
k
k
g S S (mod p) (g 1)S 0 (mod p)

Como g e uma raiz primitiva e (p 1) k temos que g k 1 6 0 (mod p), assim pelo cancelamento obtemos
S 0 (mod p), o que encerra a prova.
Exemplo 94. Mostre que 2 e uma raiz primitiva m
odulo 3k para todo k 1.
Soluca
o. Temos que mostrar que ord3k 2 = (3k ) = 2 3k1 . A prova e por inducao em k. O resultado
e verdadeiro para a base k = 1. Suponha agora que o resultado valha para k e seja d = ord3k+1 2. Pelo
menor divide temos que d | (3k+1 ) d | 2 3k . Por outro lado, como 2d 1 (mod 3k+1 ) = 2d 1
(mod 3k ) e ord3k 2 = 2 3k1 por hip
otese de inducao, temos tambem que 2 3k1 | d. Assim, so h
a duas
k1
possibilidades para d: ou d = 2 3
ou d = 2 3k . Devemos mostrar que a primeira n
ao ocorre, ou seja,
k1
que 223
6 1 (mod 3k+1 ).
k1
k1
Mas isto e um trabalho para o lema de Hensel! Pelo lema, temos que 3k k 223
1, assim 223
6 1
(mod 3k+1 ), o que encerra a prova.

45

O lema a seguir mostra como, a partir de uma raiz primitiva, encontrar todas as demais.
Lema 95. Seja g uma raiz primitiva m
odulo m. Mostre que as razes primitivas de m s
ao exatamente as
potencias g i de g com (i, (m)) = 1. Em particular, m possui ((m)) razes primitivas (se possuir alguma).
Soluca
o. Como qualquer elemento de (Z/mZ) e uma potencia de g, temos que as razes primitivas de
m tambem se escrevem como g i para algum i e a quest
ao se resume a provar que ordm g i = (m)
(i, (m)) = 1.
Seja d = (i, (m)) e t = ordm g i . Vamos provar que t = (m)/d, o que implica o lema. Primeiramente,
observe que como i/d Z temos
i/d
(m)/d
=1
= g (m)
gi
Assim, pelo menor divide temos que t divide (m)/d. Para mostrar que, reciprocamente, (m)/d divide
t, observe que como
t
g i = 1 g it = 1
|
e g e uma raiz primitiva, novamente pelo menor divide temos que (m) | it (m)
d
(m) i
(m)
( d , d ) = 1, pela alternativa de primo temos que d | t, completando a demonstracao.

4.4

i
d

t. Como

Demonstrac
ao do Teorema Principal

Vamos agora demonstrar o teorema acima. Comecamos com uma pequena digress
ao sobre polinomios sobre
corpos finitos.

4.4.1

Polin
omios com coeficientes em Z/pZ

Um corpo e um anel em que a multiplicacao e comutativa e em que todo elemento n


ao nulo possui inverso
multiplicativo. Por exemplo, Q, R e C sao corpos, mas Z n
ao. Uma quest
ao natural e: quando Z/mZ e um
corpo?
Lema 96 (Corpo Finito). Z/pZ e um corpo se, e somente se, p e primo.
Demonstraca
o. Se p e primo, ent
ao todo elemento com excecao de 0 e inversvel, logo Z/pZ e um corpo.
Reciprocamente, se Z/pZ e um corpo, os n
umeros 1, 2, . . . , p 1 devem ser todos inversveis modulo p, isto
e, primos com p. Isto implica que p e primo (c.f. demonstracao do teorema de Wilson).
Quando trabalhamos em um corpo K podemos fazer a divisao euclidiana de polinomios com coeficientes
em K utilizando o algoritmo da chave e o fato de que qualquer coeficiente n
ao nulo ser inversvel. Por exemplo,
dividindo o polinomio x3 + 3x + 2 por 2x + 1 com coeficientes em Z/5Z obtemos quociente 3x2 + x + 1 e
resto 1:
2x + 1
x3 +0x2 +3x+2
x3 3x2
3x2 + x + 1
2x2 +3x+2
2x2 x
2x+2
2x1
1
Teorema 97. Seja f (x) um polin
omio com coeficientes inteiros e de grau d e p um primo. Ent
ao a congruencia f (x) 0 (mod p) tem no m
aximo d razes m
odulo p, contando multiplicidades.

46

Demonstraca
o. Inducao em d. Para d = 0, n
ao h
a razes e para d = 1, o polinomio e da forma ax + b, a 6 0
(mod p), cuja raiz e x = b a1 mod p.
Seja f (x) de grau d e r uma raiz de f (caso f n
ao tenha razes, o teorema est
a demonstrado, pois d 0).
Ent
ao, pelo teorema do resto, f (x) (x r)g(x) (mod p), sendo g de grau d 1. Pela alternativa de primo,
temos que (x r)g(x) 0 (mod p) x r (mod p) ou g(x) 0 (mod p). Pela hipotese de inducao, g
tem no maximo d 1 razes, e o resultado segue.
Note que esse resultado n
ao e valido para modulos compostos. Por exemplo, x2 1 0 (mod 8)
tem 4 solucoes, a saber, todos os mpares modulo 8.

4.4.2

Enfim, a demonstrac
ao!

Vamos provar o teorema 92 em varias partes.


Lema 98. Se m tem dois fatores primos mpares distintos p e q ent
ao m n
ao admite raiz primitiva.
Demonstraca
o. Se m admite dois fatores primos mpares distintos p e q ent
ao podemos escrever m = a b,
com (a, b) = 1, a, b > 1, p | a e q | b. Note que p 1 | (a) e q 1 | (b). Em particular, (a) e (b)
sao ambos pares, ou seja, (a)/2 e (b)/2 sao ambos inteiros. Assim, sendo x primo com m, x(a) 1
(mod a) = x(a)(b)/2 1 (mod a). Analogamente, x(a)(b)/2 1 (mod b). Como a e b sao primos entre
si, conclumos que x(a)(b)/2 1 (mod ab), de modo que ordm x (a)(b)/2 = (ab)/2 = (m)/2 < (m);
ou seja, m n
ao admite raiz primitiva.
Lema 99. 2 e 4 admitem raiz primitiva, mas 2n , n 3, n
ao.
Demonstraca
o. Primeiro, 1 e 3 sao razes primitivas de 2 e 4, respectivamente. Seja x mpar. Temos x2 1
(mod 8) e, para n 3,
n2
n3
x2
1 = (x2 1)(x2 + 1)(x4 + 1) . . . (x2
+ 1)
i

n2

tem pelo menos n fatores 2, j


a que 23 | x2 1 e 2 | x2 + 1, i = 1, 2, . . . , n 3. Logo x2
1 (mod 2n ), ou
n2
n
n
< (2 ) para todo x mpar. Assim, 2 n
ao admite razes primitivas para n 3.
seja, ord2n x 2
Lema 100. Se 4 | m e m > 4 ent
ao m n
ao admite raiz primitiva.
Demonstraca
o. Seja m = 2k , mpar e k 2. O caso = 1 ja foi estudado no lema anterior. Nos demais
casos, basta repetir a demonstracao do primeiro lema com 4 no lugar de p e q sendo um divisor de .
Lema 101. Todo primo mpar p admite raiz primitiva.
Demonstraca
o. Essa e a parte mais difcil do teorema e e aqui que utilizaremos o teorema sobre polin
omios mod p. Considere o seguinte algoritmo para encontrar uma raiz primitiva de qualquer primo p:
Algoritmo
1. Tome a = 2.
2. Seja da = ordp a. Se da = p 1, a e raiz primitiva de p; caso contrario, tome o menor n
umero b que
n
ao e congruente a algum ai mod p.
3. Seja db = ordp b. Se db = p 1, b e raiz primitiva de p; se n
ao, tome m e n tais que (m, n) = 1, m | da ,
n | db e mn = mmc(da , db ) (por que eles existem?).
4. Troque a por c = ada /m bdb /n e volte ao passo 2.

47

Vamos provar que esse algoritmo funciona e, o mais importante, termina. Supondo que termine, o
algoritmo funciona porque ele so para quando encontramos uma raiz primitiva.
Agora, provemos que o algoritmo termina, o que e mais interessante. Primeiro note que db n
ao divide
ao pode ocorrer pois a equacao xda 1 (mod p)
da , pois se dividisse teramos bda 1 (mod p), o que n
admite no maximo da solucoes, que sao 1, a, a2 , . . . , ada 1 , e b n
ao e congruente a nenhum ai . Isso implica
da /m db /n k
mmc(da , db ) > da . Alem disso, seja k = ordp c. Ent
ao (a
b
) 1 (mod p). Elevando ambos os
membros por m, obtemos ada k bdb mk/n 1 (mod p) bdb mk/n 1 (mod p). Lembrando que db = ordp b,
pelo menor divide db | db mk/n n | mk. Sendo (m, n) = 1, temos n | k. Analogamente, m | k e,
portanto mn | k. Observando ainda que cmn 1 (mod p), temos ordp c = mn = mmc(da , db ) > da . Isto quer
dizer quem a cada iteracao do algoritmo a ordem do proximo valor aumenta. Portanto, em algum momento
iguala o seu maximo, que e p 1 (note que a escolha de b no algoritmo depende de ordp a 6= p 1).
Lema 102. Se g e raiz primitiva de p mas n
ao de p2 , ent
ao g + p e raiz primitiva de ambos.
Demonstraca
o. Seja d = ordp2 g. Ent
ao
gd 1

(mod p2 ) = g d 1

(mod p) = p 1 | d

Logo, como d 6= p(p 1) e d | p(p 1), d = p 1, isto e, g p1 1 (mod p2 ).


2
Basta agora demonstrarmos que (g + p)p1
 6 1 (mod 2p ). E esse e um trabalho para o binomio de
p1 p2
p1
p1
Newton! Temos (g + p)
g
+ 1 g
p (mod p ) (voce consegue ver por que n
ao precisamos
escrever os demais termos do bin
omio de Newton?). Substituindo g p1 1 (mod p2 ) e desenvolvendo
obtemos (g + p)p1 1 + (p 1)pg p2 1 pg p2 (mod p2 ), que n
ao e 1, pois g p2 6 0 (mod p). Logo
2
g + p e raiz primitiva de p (e de p tambem!)
Lema 103. Se g e raiz primitiva de p e p2 ent
ao e raiz primitiva de pn , e portanto pn admite raiz primitiva.
Demonstraca
o. Inducao sobre n. A base de inducao (n = 1 e n = 2) est
a na hipotese. Suponha que g seja
ao
raiz primitiva de pn1 . Seja d = ordpn g. Ent
gd 1

(mod pn ) = g d 1

(mod pn1 ) = ordpn1 g | d = pn2 (p 1) | d

Como d | (pn ) d | pn1 (p 1), temos d = pn2 (p 1) ou d = pn1 (p 1). Para eliminar o primeiro
caso, vamos usar o lema de Hensel.
Note que como p | g p1 1 e p2 n
ao divide g p1 1 (caso contrario, g n
ao seria raiz primitiva de p2 ),
n2
p1
pn2 (p1)
pkg
1. Assim, pelo lema de Hensel, a maior potencia de p que divide g
1 = (g p1 )p
1
n2
n2
e p1+n2 = pn1 . Assim, pn n
ao divide g p (p1) 1, o que e equivalente a g p (p1) 6 1 (mod pn ). Logo
ordpn g = pn1 (p 1) = (pn ) e, portanto, g e raiz primitiva de pn .
Lema 104. 2pn admite raiz primitiva.
Demonstraca
o. Seja g uma raiz primitiva de pn . Note que (2pn ) = (pn ) (verifique!) e considere g ou
n
g + p , o que for mpar, e denote-o por h. Sendo d a ordem de h, temos hd 1 (mod 2pn ) = hd 1
(mod pn ) = (pn ) | d. Assim, d = (2pn ).
Ufa! Com isto, o teorema est
a provado!

4.5
4.5.1

Exerccios
B
asicos

49. Encontre a ordem de


(a) 11 mod 190
(c) 5 mod 1024

(b) 7 mod 123


(d) 2 mod 343
48

50. Encontre todas as razes primitivas de


(a) 11
(c) 10

(b) 23
(d) 50

51. Mostre que


(a) 2 e uma raiz primitiva de 11;
(b) 7 e uma raiz primitiva de 239;
(c) 2 e uma raiz primitiva de 5k para todo k.
52. Sejam x e y inteiros positivos. Prove que existem inteiros positivos m e n tais que m | x, n | y,
(m, n) = 1 e mn = mmc(x, y).
53. Prove o item (ii) do lema de Hensel.

4.5.2

Lic
ao de Casa

54. (IMO) Os tres u


ltimos dgitos de 1978m s
ao iguais aos tres u
ltimos dgitos de 1978n (1 m < n,
m, n N ). Determine m e n tais que m + n seja mnimo. (Cuidado! A resposta n
ao e 1 + (1000)!!)
n

55. Mostre que se p e um primo tal que p | 22 + 1 ent


ao p 1 (mod 2n+1 ). Utilize este fato para encontrar
25
um fator primo de 2 + 1.
56. Sendo k 2 e n1 , n2 , . . . , nk 1 n
umeros naturais tais que n2 |2n1 1, n3 |2n2 1, . . . , nk |2nk1 1 e
nk
n1 |2 1, mostre que n1 = n2 = = nk = 1.
57. (OBM) Encontre todas as funco
es f : Z>0 R tais que, para todos x, y inteiros n
ao negativos,
f (x)f (y) = f (xy)

f (x + 1019) = f (x)

58. Seja f (x) um polin


omio de coeficientes inteiros com grau d. Prove que a congruencia f (x) 0 (mod p),
p primo, tem d soluco
es n
ao congruentes entre si mod p se, e somente se, na divis
ao euclidiana de xp x
p
por f (x), x x = f (x)q(x) + r(x), o resto r(x) tiver todos os seus coeficientes m
ultiplos de p.
59. Sendo k um inteiro positivo dado, encontre todos os inteiros positivos n tais que 7n + 1 e m
ultiplo de 5k .
60. H
a outra maneira de provar que p admite raiz primitiva. Como? Siga os itens!
P
(a) Prove que d|n (d) = n. Dica: conte de duas maneiras a quantidade de pares (x, d) em que (x, n) = d.
(b) Prove que se d | p 1 a congruencia xd 1 (mod p) tem exatamente d soluco
es distintas mod p.

(c) Seja d um divisor de p 1 e r(d) a quantidade de n


umeros mod p com ordem igual a d. Prove que
r(d) (d).
(d) Prove que, na verdade, r(d) = (d). Em particular, p admite (p 1) razes primitivas.

4.5.3

Quest
oes de Prova

61. Encontre todos os n


umeros inteiros positivos n tais que x25 x (mod n) para todo inteiro x.
62. (IMO) Determine se existe um inteiro n com 2000 divisores primos distintos tal que n divide 2n + 1.

49

63. Encontre todos os inteiros m, n e p, onde p e um primo mpar tais que


pm np = 1
Dica: tente encontrar uma fatoracao e calcule o mdc dos dois fatores da fatoracao. Depois use o binomio de
Newton para concluir o problema.
64. (China) Encontre todos os inteiros n
ao negativos x, y, z e w tais que
2x 3y 5z 7w = 1
65. (IMO) Determine todos os inteiros n 1 tais que (2n + 1)/n2 seja inteiro.
66. (IMO) Determine todos os pares (n, p) de inteiros estritamente positivos tais que p e primo, n 2p e
(p 1)n + 1 e divisvel por np1 .
67. (Banco IMO) Encontre todas as ternas (a, m, n) de inteiros positivos tais que am + 1 divide (a + 1)n .
68. (Banco IMO) Seja a > 1 e m > n inteiros positivos. Prove que se am 1 e an 1 tem os mesmos
divisores primos ent
ao a + 1 e uma potencia de 2.
69. (IMO Shortlist) Seja p > 3 um primo. Prove que existe a com 1 a < p 1 tal que ap1 1 e
(a + 1)p1 1 n
ao s
ao divisveis por p2 .

50

Você também pode gostar